Open navigation menu
Close suggestions
Search
Search
en
Change Language
Upload
Sign in
Sign in
Download free for days
0 ratings
0% found this document useful (0 votes)
1K views
62 pages
Goljan's High-Yield Facts
For usmle step1 preparation.
Uploaded by
Taimur Jahangir
Copyright
© © All Rights Reserved
We take content rights seriously. If you suspect this is your content,
claim it here
.
Available Formats
Download as PDF or read online on Scribd
Download
Save
Save Goljan's High-Yield Facts For Later
0%
0% found this document useful, undefined
0%
, undefined
Embed
Share
Print
Report
0 ratings
0% found this document useful (0 votes)
1K views
62 pages
Goljan's High-Yield Facts
For usmle step1 preparation.
Uploaded by
Taimur Jahangir
Copyright
© © All Rights Reserved
We take content rights seriously. If you suspect this is your content,
claim it here
.
Available Formats
Download as PDF or read online on Scribd
Carousel Previous
Carousel Next
Download
Save
Save Goljan's High-Yield Facts For Later
0%
0% found this document useful, undefined
0%
, undefined
Embed
Share
Print
Report
Download now
Download
You are on page 1
/ 62
Search
Fullscreen
‘Note: This material is copyrighted. All rights reserved. Edward Goljan, M.D. 2003 INTEGRATED PATHOLOGY NOTESis copyrighted. All rights reserved. Edward Goljan, M.D. 2003 INTEGRATED PATHOLOGY NOTES ° ‘The following are high yield facts that are meant to complement the notes. Some of the items cover concepts that are taught by other teachers in the course that directly relate to pathology. Other topics that I do not have time to teach during the review (e.g., environmental pathology, nutrition, and immunopathology). Some items are clinical scenarios on high yield subjects that T use on exams and quizzes for my own students that will provide you with additional questions to quality control what you have leaned. © Terms for lab test results; « true positive (TP)- positive test result in a patient withthe disease, « false negative (FN) negative (normal) test result in a patient with the discase, « true negative CEN)- negative (normal) test result in a patient without the disease, + false positive (FP)- positive test result in @ patient without the disease, « negative predictive value of a test result (PV)- chance that a negative (normal) test result is a TN rather than a FN, positive predictive value of a test result (PV")— chance that a positive test result is a TP rather than a FP © Sensitivity of a test: 1. "positivity in disease" formula for sensitivity: TP /TP-+ FN 2, use of a test with 100 % sensitivity » best used to screen for disease, « excludes disease when the test result returns negative, « includes people with disease when the test result seturns positive, « buzz words: excludes and includes 3. megative predictive value (PV-) of a test with 100% sensitivity- « PV" must be 100% since there are no FN test results in a test with 100% sensitivity~ this underscores why tests witb 100% sensitivity exclude disease when the test result returns negative, * eg., serum antinuclear antibody test (ANA) has 100% sensitivity in patients with SLE, therefore repeatedly negative serum ANAs excludes SLE SLE No SLE Positive test (TP) 100 (FP) 20 Negative test (FN) 0 (EN) 80 Sensitivity of ANA: TP / TP +FN= 100 / 100 +0 = 100% PV": TN/ TN-+ FN = 80/80 +0= 100% positive predictive value (PV") of a test with 100% sensitivity— « test result is either a TP or FP, therefore, the PY* is never 100% owing to the chance that it may be a FP, « people swith the disease are always included, « e-g., a positive serum ANA result includes all people ‘with SLE, however, it does not confirm SLE since other discases also have a positive serum ANA (c.,, rheumatoid arthritis, progressive systemic sclerosis) SLE NoSLE Positive test (TP) 100 &P) 20 Negative test (FN) 0 (EN) 80 Py! =P /IP +FP= 100 / 100 +20=83% & Specificity of a test: 1, “negativity in health" formula for specificity is TN /TN + FP 2. use of a test with 100% specificity confirms disease since there are no FP test results 3. PV" ina test with 100% specificity— * PY" is always 100%, since the FP sate is zero this underscores why test with 100% specificity are used to confirm disease, * ¢.g., anti-Sm for SLE has 100% specificity (no FPs) and 30% sensitivity all patients with a positive anti-Sm have SLE SLE —_-NoSLE Positive anti-Sm (mP)30 FP) 0 Negative anti-Sm (FN) 70 (TN) 100‘Note: This material is copyrighted. All rights reserved. Edward Goljan, M.D. 2003 PV':= TP/ TP + FP = 30/30 +0 = 100% 4, PV in a test with 100% specificity-* PV" may be a TN or EN- « it does not exclude i anti-Sm is negative in a patient * does not exclude SLE, * use other tests to confirm SLE if your suspicions are high e.g., anti-double stranded DNA SLE No SLE Positive anti-Sm (TP) 30 FP) 0 Negative antiSm (FN) 70 (TN) 100 PV =TN/TN +EN= 100 / 100 +70 = 59% = — How to calculate the reference interval of the test when given the mean of the test and 1 SD (standard deviation): 1. remember to double the SD- 2 SD covers 95% of the normal population 2. example nan ofthe test= 100 mg/dL and 1 SD = 5 mg/dL (2 SD = 10 me/aL) B, reference interval = 90-110 mg/dL (100 - 10= 90 and 100 + 10 = 110) 3. for each test, S% of normal people will have test results outside the reference interval— ‘A. chance of a FP incrcases when mare than one testis ordered on a patient B. example, 2 tests on a patient increases the chance of a FP test result on one of those tests to ~10% 4, SD isa marker of the precision (reproducibility) of the test~ it is not a marker of how accurate the test result is ©) Accuracy: good ‘Accuracy: poor Precision: good Precision: good = Effect of test sensitivity/specificity of a test on prevalence: 1, test with highest sensitivity (not specificity) increases prevalence of disease (number of people in a population that have disease)- ‘A. it detects more people with the disease since itis a good screening test B. tests with high specificity confirm disease and help differentiate a TP from a FP but they are poor screening tests % Effect of increasing the upper limit of normal of a test reference interval (e.g. raising a reference interval of 0-4 ng/mL to 0-10 ng/mL) on sensitivity, specificity, PV", and PV: 1. increases specificity and positive predictive value~ ‘A. higher values are mare likely to represent TPs than FPs B. specificity always increases, which automatically increases PV" 2, decreases sensitivity and negative predictive value (PV) ‘A. increasing specificity of a test always decreases its sensitivity and PV- B. _ FN rate increases, since more people with disease are encountered as the reference interval increases C. anormal test result is more likely to be a FN rather than a TN(Note: This material Is copyrighted. All rights reserved, Edward Goljan, M.D. 2003 Effect of decreasing the upper limit of normal of a test reference Interval (eg, lowering the fasting glucose level for diagnosing dabetes elites [DM] frm >140 m/l. >126 mglaL) on sensitivity, specificity, PV’, and PV: 1 increases sensitivity and negative predictive value (PV) - ‘A. dropping the upper limit to a lower value means that more people with a negative test result are likely to be TNs (not have DM) rather than FNs B. sensitivity and PV” always increase when the upper limit of a testis lowered decreases specificity and positive predictive value (PV")- ‘A. fewer people arc likely to have DM, a test result >126 mg/L. is more likely to be a FP than a TP test result B. summary schematic Normal Disease 0 — Interval 0-4 Sensitivity 100% (no FNs) PV" 100% Specificity decreases PV" decreases —_—> Interval 0 ~ 10 Specificity 100% (no FPs) PV* 100% Sensitivity decreases PV" decreases 10 Prevalence: 1. Prevalence (number of people with disease in the population studied) = Incidence (aumber of new cases over a period of time) x Duration of the disease— P=IxD as duration (D) decreases, prevalence (P) decreases, as D increases, P increases incidence (1) is a constant in this relationship PV+ inreases when prevalence increases: more TPs PV- increases when prevalence decreases: more TNs prevalence calculation TP + FN (all people with diseasey/ TP + FN + TN + FP (all people with and without disease) example— if treatment for leukemia lengthens the survival period but does not lead to its cure, prevalence (P) of leukemia increases owing to the increase in duration (D): no effect on incidence (number of new cases of leukemia) BmOOR>‘his material is copyrighted. All rights reserved. Raward Goljan, M.D. 2003 © Example of a calculation for sensitivity, specificity, PV+, PY-, and prevalence: Disease No Disease Positive test (TP) 60 (RP) 40 Negative test (FN) 20 (TN) 80 Sensitivity of the test: TP / TP + FN= 60 / 80= 75% Specificity of the test: TN /'TN + FP = 80 / 120 = 66% PV-: TN/ TN + EN = 80 / 100 = 80% (80% chance it is a TN and a 20% chance it is a FN) PV": IP / TP + FP = 60 / 100 = 60% (60% chance it is a TP and 40% chance it is a FP) Prevalence: TP + FN / TP + FN + TN + FP = 80 /200 = 40% = Normal changes in pregnancy: 1. greater increase in plasma volume than RBC mass— A. decreases hemoglobin (Hb) and hematocrit (Het): dilutional effect B. increases glomerular filtration rate (GFR) and creatinine clearance (Cr): due to increased plasma volume C. decreases serum BUN/creatinine/urie acid: dilutional effect + increased clearance 2, imereased alkaline phosphatase— placental origin 3. respiratory alkalosis estrogen/progesterone effect on CNS respiratory center causing increased clearance of CO, per breath 4, increased T, and cortisol- ‘A. _ increased synthesis of their respective binding proteins B. free hormone levels are normal C. no signs of hyperthyroidism/hypercortisotism D. eg, normal serum TSH and ACTH, respectively, © Newborn: high hemoglobin (Hb) due to increase in HbF— i. eft shifts oxygen dissociation curve (ODC): causes tissue hypoxia—> 2. stimulus for erythropoietin (ERO) release—> 3. increases RBC production with subsequent increase in Wb concentration = Children: 1, Increased seram alkaline phosphatase (ALP)~ ‘A. 3-5 times higher than adults B. osteoblasts release enzyme when stimulated by vitamin D C. ALP increases bone mineralization 2. imereased serum phosphate- required to drive calcium into bone 3, _ slight decrease in hemoglobin concentration when compared to adult levels = Main laboratory difference in adult male and female: 1. iron studies are all lower in women- ©.g,, Serum iron and ferritin 2. lower Hb concentration in women = Changes in old age: 1. renal changes decrease in GFR and CCr 2. slight inerease tm alkaline phosphatase duc to reactive bone formation in osteoarthritis 3. slight decrease in Hgb concentration in males— due to decrease in androgens (normal values are those of a female) = Analytes increased with hemolyzed blood sample secondary to venipuncture: 1 LDH A. LDB, isoenzyme fraction is primarily increased and is greater than LDH; isoenzyme fraction (LDIH/LDEb flip)Note: This material is copyrighted. Ail rights reserved. Edward Goljan, M.D. 2003, B, false positive acute myocardial infarction C. LDH; isoenzyme is also in cardiac muscle LDH, LDH; LDH, LDH. LDH, Normal LDH isoenzymes LDH, LDH, LDH, LDH, LDH; Acute myocardial infarction: LDHy/LDE; flip 2. potassium— ‘A. pseudohyperkalemia B. Kris the major intracellular cation C. ECG will not show a peaked T wave Lipid most affected by fasting: 1. triacylglycerol (TG) component coming from chylomicrons chylomicrons contain diet- derived TG 2. fasting or lack of fasting does not affect cholesterol (CH) and high-density lipoprotein (ADL) concentration— A. normally, CH is < 3% of the chylomicron fraction B. fasting is unnecessary for an accurate CH or HDL-CH 3. fasting is necessary for an accurate calculated low-density lipoprotein (LDL)~ A. LDL=CH-HDL-TG/S5 (VLDL fraction) B. if TG is falsely increased by chylomicrons from the diet, it will falsely lower the calculated LDL and falsely increase VLDL (TG/5) Laboratory test alterations in alcoholics: 1, enhancement of the liver cytochrome P-450 system ‘A. increased synthesis of y- glutamyltransferase (GGT) B. excellent enzyme marker for alcoholic fiver disease ‘reased production of NADH in its metabolic breakdown causes biochemical reactions involving NADH to move in ts direction resulting in the following— ‘A. lactic acidosis: pyruvate-> lactate B. fasting hypoglycemia: pyruvate is unavailable for gluconeogenesis CC. hypertriacylglycerolemia: 1,3 bisphosphoglycerate> dihydroxyacetone phosphate—> slycerol 3-phosphate—> TG 3. imerease in ketoacid synthesis ‘A. acetyl CoA, the end product of alcohol metabolism is used in the following reaction B. acetyl CoA + acetyl CoA > C.acetoacetyl CoA»Note: This material is copyrighted. Al rights reserved. Edward Goljan, M.D. 2003 D. HMG CoA-> acetoacetic acid: increase in NADH converts it into B hydroxybutyric acid (B-OHB) 4, imerease in fatty acid synthesis due to the inerease in acetyl-CoA. 5. hyperuricemia lactic acid/ketoacids compete with uric acid for excretion in the kidneys 6. imereased anion gap metabolic acidosis lactate + -OHB Laboratory test alterations in smokers: 1. respiratory acidosis air gets in but cannot get out, so COr is retained 2. hypoxemia (low PaO,)~ increase in PCO, always causes a decrease in PO; 3. decreased O, saturation decrease in PaO; always decreases SaO 4, imereased carbon monoxide (CO) levels CO is present in cigarette smoke 5, secondary polycythemia low PaO; stimulates erythropoietin release and an increase in the production of RBCs 6. absolute neutrophilic leukocytosis~ metabolites in smoke mobilize the neutrophil ‘rsarginating pool in the circulation by decreasing leukocyte adhesion to endothelial cells, = Lyon's hypothesis: 1. one of the 2 X chromosomes in a female is randomly inactivated © ~S0% X chromosomes are maternal, ~50% X chromosomes paternal, « inactivated X chromosome ‘becomes a Barr body- projection from the nucleus counted in squamous cells obtained by scrapings from the buccal mucosa 2. normal females have 1 Barr body and normal males do not have Barr bodies 3, male with Klinefelter syndrome (XXY genotype) has 1 Barr body 4. female with Turner's syndrome (XO) has no Barr bodies F Trinucleotide repeat disorders: + progressively worse disease occurs in future ‘generations (phenomenon is called anticipation), * constant repetition of 3 nucleotide bases (¢.g., CAG, CAG, CAG etc), * more trinucleotide repeats are added each generation leading to progressively worse disease and earlier manifestation of the disease, * examples Huntington's disease (AD), fragile X syndrome (SXR, female carriers with trinucleotide repeats eventually ‘become symptomatic), Friedreich's ataxia (AD), myotonic dystrophy (AD) = — Nondisjunction: « cause of the majority of chromosome number disorders (e.g., trisomy 21, ‘Tumer’s XO), ¢ due to unequal separation of chromosomes in first phase of meiosis = — Mosaicism: nondisjunction of chromosomes in mitotic division in the early embryonic period, © two chromosomally different cell lines are derived from a single fertilized egg, * most cases involve sex chramosomes- e.g., gonadal dysgenesis with XO/KX, XO/KY & Translocation: ¢ one part of a chromosome is transferred to a non-homologous or homologous chromosome, « called a balanced translocation ifthe translocated fragment is functional, Robertsonian translocation in Down syndrome: type of balanced translocation with reciprocal translocation between 2 acrocentric chromosomes (™O=mmmmm), # usually chromosomes 21 and 14, + produces 1 long chromosome (I4*21): extremely small translocated chromosome usually lost, * mother has the translocation: 43 chromosomes with 1 long #1421, 1 normal #14, and 1 normal #21, * mother is normal, since both translocated fragments are functional, * Down child receives 1 normal #21 from uninvolved parent (father) and 1 14¢21 and 1 normal #21 from affected mother for a total of 46 chromosomes but all 3 #21s are functioning 2 al 4 4 yD “4 | ut | I parent: 45 chromosomes Down syndrome: 46 chromosomes‘Note: This material is copyrighted. All rights reserved. Edward Goljan, M.D. 2003, Cri-du-chat: © deletion of short arm of chromosome 5, + mental retardation, » ery like a cat ‘Microdeletion syndromes: « loss of a small portion from 1 chromosome can only identified with fhigh resolution techniques, * microdeletion on chromosome 15 may result in the Prader-Willi syndrome + chromosome 15 deletion is of paternal origin, obesity, + hypogonadism, mental retardation, © microdeletion on chromosome 15 may result in Angelman syndrome~ + chromosome 15 deletion is of matemal origin, * "happy puppy" syndrome (child always happy/laughing but cannot talk), « term applied to these syndromes is genomic imprinting © — Genetics disorders in African-Americans: « sickle cell trait/disease— 810% prevalence of sickle cell gene, * a-/B-thalassemia, + glucose G-phosphate dehydrogenase (GEPD) deficiency, « hereditary persistence of HgbF & Genetic disorders in Ashkenazi Jews: « factor XII deficiency, « Gaucher's disease, « Tay-Sachs disease © — Genetic disorder in Northern Europeans: cystic fibrosis~ MC genetic disease interfering with the patient's ability to reproduce owing to early death or problems with fertility © — Genetic disorders in Mediterranean peoples: « GOPD deficiency, + sickle cell trait thalassemia Genetic disorder in Southeast Asians: o-thalassemia [MC genetic syndrome associated with advanced maternal age: trisomy 21 Down syndrome pathogenesis: « trisomy 21— + 95% of all cases, * 47 chromosomes, + maternal origin for extra chromosome, Robertsonian translocation- * 4%, * 46 chromosomes in child, # mother with 45 chromosomes usually mother, * MC genetic cause of mental retardation— 1Q 25-50 in 80% = Down's clinical findings: « epicanthal folds with upward slanting, « simian palmar crease, * poor reflexes/hypotonicity, « cardiovascular— + endocardia! cushion defects (combined ASD and VSD), ‘sor determining factor for suosalin cai infancy and-childhoo, « Gl + duodenal asia (polyhydramnios, vomits bile at birth, double bubble sign on x-ray), + Hirschsprune’s disease , Tenatologie~ inreased incidence of leukemia, « CNS-» Alzheimer's disease,» chromosome 21 codes for f-amyloid proteins, which are converted into amyloid and are toxic to neurons, * aumiversal by age 35 (any patient with Alzheimer’s disease under 40 is a patient with Down a4 ase, © Be 9494 syndrome), « reproductive~ * all males ate sterile, + females have $0% chance of having a child with Down's = — Risk for future children with Down’s: © 1-2% overall risk for trisomy 21, « matemal age~ women >35 yrs of age, # 5-15% risk for parent with a balanced translocation, * karyotype of affected child should always be determined to evaluate risk for siblings to have affected children Mendelian disorders in descending order of frequency: « autosoma! dominant (AD), autosomal recessive (AR), # sex-linked recessive (SKR), # sex-linked dominant (SXD) = AD disorders: « only 1 abnormal allele is necessary to express the disease- * "dominant gene”, * 8. aa (disease, usually lethal) or Aa (disease), « only one parent has to have the gene to pass it on children, * one affected parent + normal parent- 50% normal children, 50% affected srders: « both abnormal alleles must be present (homozygous) to express the disease~ * aa (disease), * Aa (heterozygote asymptomatic carrier), « both parents must have the abnormal allele~ * 2 asymptomatic carriers (children- 25% notmal AA, 50% Aa, 25% aa), * patient with the disease thas children with an asymptomatic carrier (children 50% Aa, 50% 2a) & —SXR disorders: « males with abnormal allele express disease- males are "homozygous", since they only have 1X chromosome, « affected males transmit the disease to both daughters but none of their sons— daughters are usually asymptomatic carriers, « female carrier transmits the disease to 50% of the boys and 25% of daughters are asymptomatic carriers, « female carrie + affected male—Note: This material is copyrighted. AM rights reserved. Edward Goljan, M.D. 2003 0% of males are affected, 25% of females are asymptomatic carriers, 25% of females ace ‘symptomatic maternal and paternal X chromosome are abnormal SXD disorders: males and heterozygous females both express the disease AD disorders: # associated with structural defects in proteins and receptors— + enzyme deficiencies fare uncommon, * AD disorders that are enzyme deficiencies~ acute intermittent. porphyria (deficiency of uroporphyrinogen synthase), * hereditary angioedema (C1 esterase inhibitor deficiency), « late manifestations of disease ¢.g., Huntington's disease with chorea/dementia later in life, © exhibit penetrance + person has the abnormal gene but never expresses the disease, however, the gene is transmitted to their children, * some AD disorders have 100% penetrance (@g,, familial polyposis, adult polycystic kidney disease), while others have incomplete penetrance (c.g, Marfan's), « exhibit variable expressivity~ affected people have different levels of severity of the disease, ¢ mechanisms of AD disease without a family Hx: * MCC is incomplete penetrance, + new mutation, ¢ examples of AD diseases (in order of decreasing frequency): * von Willebrand's disease + familial hypercholesterolemia, adult polycystic kidney disease, hypertrophic cardiomyopathy, + Huntington's disease, * neurofibromstosis, * congenital spherocytosis, * familial polyposis, * acute intermittent porphyria, + osteogenesis imperfecta, » Marfan syndrome © Example of a pedigree with complete penetrance: a4 —-O x O & Example of a pedigree with incomplete penetrance: 7 '& AR diseases: + na evidence af- penetrance, variable expressivity, late manifestations, * most (not all) AR diseases are enzyme deficiencies- inborn errors of metabolism~ * acute intermittent porphyria and Cl esterase inhibitor deficiency are both autosomal dominant, * G6PD and Lesch- Nyhan are both SXR, » examples of thase that are nat enzyme deficiencies + cystic fibrosis, + sickle cell traiv/disease, * hemochromatosis, * Wilson's diseaseNote: This material is copyrighted. All rights reserved. Edward Goljan, M.D. 2003 & Example of an AR pedigree: _[-+—O oa © —SXR disorders: examples (in order of decreasing frequency): * fragile X syndrome, » G6PD deficiency, * Duchenne's muscular dystrophy, * hemophilia A/B, + severe combined immunodeficiency, * Wiskott-Aldrich syndrome, * testicular feminization, + color blindness, * chronic granulomatous disease of childhood, *Bruton’s sgammaglobulinemia 6 6 c O ano of O GO & Fragile X syndrome: « some geneticists say it is a sex-linked dominant disease, owing to the fact, that female carriers may express the disease in future generations (concept of anticipation), * second MC genetic cause of mental retardation, * MC Mendelian disorder associated with mental retardation, * clinical~ + mental retardation, + macroorchidism- at puberty, + ~30% of female carriers are mentally retarded or have impaired leaming— due to anticipation and addition of trinucleotide repeats with future ions, + abnormal fragile X chromosome- DNA analysis for carrier identification (identify CGG trinucleotide repeat is the best test to confirm) ‘@ —Lesch Nyhan syndrome: * SXR disease with a deficiency of HGPRT- no inhibition of PRPP in purine metabolism, « clinical- + hyperuricemia, * mental retardation, * self mutilation © SXD disorders: * percentages of children with the abnormal allele are the same as those in SXR. disorders: + dominant abnormal allele causes discase in bath males and females, + affected woman transmits symptomatic disease to 50% of her daughters and 50% of her sons, + affected males transmit symptomatic disease to all of their daughters and none of their sons, « examples: * familial ‘hypophosphatemia (defect in the proximal reabsorption of phosphate and conversion of 25(OH)D, to 1,25 (O);D,), * Alport_syndrome (hereditary glomerulonephritis associated with nerve oo OL ,oOee 0 ‘= —Maultifactorial (polygenic) inheritance: © multiple small mutations plus the effect of environment + should be suspected when there is an increased prevalence of disease among relatives of affected individuals, * parents and offspring have 50% of their genes in common, ©a4a8 his material Is copyrighted. All rights reserved. Edward Goljan, M.D. 2003, examples: + cleft lip or palate, + congenital heart disease, + coronary artery disease, + gout, + type IIDM, # essential hypertension, + open neural defects, * congenital pyloric stenosis, Mitochondrial DNA disorders: * mtDNA primarily codes for enzymes involved in mitochondrial ‘oxidative phosphorylation reactions, ova contain mitochondria~ affected women transmit symptomatic abnormal allele to all their children, « sperm lose theit mitochondria during fertilization affected males do not transmit abnormal allele to any of their children, © examples: + Leber’s hereditary optic neuropathy, * myoclonic epilepsy, « lactic acid with stroke Example of a mtDNA pedigree: [} Calculation of the prevalence of a genetic disease given the carrier rate: e.g,, cystic fibrosis (CB), with a carrier rate of CF is 1/25~ + number of couples at risk is equal to the carries rate in males x the carrier rate in females, or 1/25 x 1/25 = 1/625 couples are at risk, + risk of having a child with CF (AR disease) is 1/4, hence 1/625 x 1/4 = prevalence of ~1/2500 Simple way of calculating the cartier rate of a disease given the prevalence of a genetic disease (Hardy Weinberg equation): « reflects the distribution of « mutant gene in the population, ‘example using prevalence of CF of 1/2500 * number of couples at risk = 1/2500 + 1/4 = 1/625, « carrier rate of CF = 1N625 = 1/25, Deformations: * anatomical defects resulting from mechanical factors (extrinsic forces) that usually occur in the last two trimesters after organs have developed, * eg., oligohydramnios producing facial and limb abnormalities (called Potter's facies) Malformation: + disturbance (e.g, drugs, infection) that occurs in the morphogenesis of an organ(s), « hypospadias faulty closure of urethral folds, « epispadias due to a defect in genital tubercle, « club foot, « ventricular septal defect genesis: anlage (primordial tissue) is sbsent (e.g, renal agenesis) ‘Aplasia: anlage is present but never develops Hypoplasia: anlage develops incompletely, however, the tissue that is present is histologically normal ‘Mechanisms of teratogeus: « teratogens are most detrimental during the embryonic period + first 9 weeks of life (4th-Sth week most sensitive for tertogens), + e.g., open neural defects occur when tube normally closes between the 23rd-28th day, ¢ specific effects of some teratogens: + some interfere with formation of mitotic spindle, + interfere with production of ATP, * some interfere with gene production (e., isotretinoin effect on Hox/hedgehog genes, which are important in embryonic patterning) ‘Teratogen cocaine: + maternal effects: * hypertension, + abruptio placenta, + newborn effects: + hyperactivity, + microcephaly (MC effect), + interruption of blood flow leading to infarction (CNS, bowel, missing digits) ‘Materval diabetes mellitus and the teratogenic effects in newborns: « increased birthweight (amacrosomia, large for gestational age): + hyperinsulinism in the fetus from poor maternal glycemic control increases muscle mass (insulin increases amino acid uptake in muscle), + hyperinsulinism increases fat deposition insulin increases deposition of TG in adipose), « open neural tube defects, + cleft lip/palate, » respiratory distress syndrome: fetal hyperinsulinism in response to maternal hyperglycemia inhibits fetal surfactant production by type II pneumocytes 10‘Note: This material is copyrighted. All rights reserved. Edward Goljan, M.D. 2003 & — ‘Teratogen diethylstilhestrol (DES): * mothers received DES to prevent threatened abortion: DES interferes with the development of mllerian structures in female fetus causing abnormalities in the tubes, uterus, cervix, upper one-third of vagina, « female siblings: + vaginal adenosis is the MC abnormality and is the precursor of clear cell adenocarcinoma of the vaginalcervix, « cervical incompetence- increased incidence of spontaneous abortions, # uterine abnormalities problems ‘with implantation, « fallopian tube abnormalities— fertility problems = Fetal alcohol syndrome and teratogente effects In newborns: + MC teratogen in United States: 2:1000 live births, « clinical~ * mental retardation (MC abnormality), + intrauterine growth retardation, + maxillary hypoplasia, + microcephaly, * atrial septal defects (least common finding), + hypoglycemia at birth © — Teratogen heroin: + small for gestational age, « irritability/hyperactivity, « high pitched ery with excessive hunger, salivation, sweating, tremors, fist sucking, « temperature instability, « seizires © — Teratogen isotretinoin: + used in treating cystic acne- + must order a pregnancy test before placing a woman on the drug, * patient must be on birth control pills while taking the drug, « newborn effects (3 C's): * craniofacial abnormalities (small ears), micrognathia, cleft palate), * cardiac defects, CNS malformations (hydrocephalus, microcephaly) ‘% — Teratogen phenytoin: ¢ nail hypoplasia, * CNS abnormalities, « cleft lip/palate, « congenital heart disease @ Maternal smoking: « vasoconstrictive effects of nicotine produce placental ischemia endotheliad
atelectasis with intrapulmonary shunting» diffuse alveolar damage and initiates spasm in the bronchioles, « immediate COD in drowning is cardiac arrhythmia © Burns: « first degree bums: painful partial thickness burns (¢.g., sunburn); produce cell necrosis limited to the epidermis; heal without sear, # second degree burns: painful partial thickness bums; involve entire epidermis; form blisters within epidermis, heal without scarring unless they are deep, ¢ third degree burns: painless full thickness bums; extensive necrosis of epidermis/adnexa; extensive scarring complicated by keloid formation (propensity for squamous cell carcinoma); healing comes ftom residual epithelium at the margin: bum and from adnexal structures, + COD: infection due to most commonly to Pseudomonas aeruginosa followed by Staphylococcus aureus, « house fires: smoke inhalation MC COD; carbon monoxide (CO) and cyanide poisoning commonly occur (source of cyanide is polyurethane in upholstery) ‘% Heat injuries: » hyperthermia: core body temperature >37.2°C, » heat cramps: afebrile; common in untrained athletes/iaborers who become volume depleted; lose excess amounts of saltwater, heat exhaustion: mild elevation in core body temperature (539°C); common in athletes tra in hotfumid environment; develop severe volume depletion, stroke: core body temperatures = 41°C; people working or running on hot day; skin hotldry; profound lactic acidosis; rhabdomyolysis common, non-exertional type of heat stroke: elderly/chronically ii! patients; skin hot/dry without sweating (hypohidrosis); lactic acidosis/ rhabdomyolysis uncommon, * malignant hyperthermia: AD disease; defect in calcium release channels in the muscle sarcoplasmic reticulum; massive muscle contractions with extremely high temperatures after induction of anesthesia by halothane and suceinylcholine (muscle relaxant); RX with dantrolene; screen family members with muscle biopsy’ and caffeine/halothane contraction test on muscle = Tonizing radiation: + examples: y-rays; x-rays; particulate radiation emitted by radioactive substances (
granulocytes > plateiets > mature RBCs); germinal cells in the testes, ovaries, « tissues with low radiosensitivity: brain is most resistant to radiation; bone; mature cartilage; muscle, + total body irradiation: hematopoietic system first system affected (greatest iB exertional type of heat 13Note: This material is copyrighted. All rights reserved. Edward Goljan, M.D. 2003, aa a4 overall effect-> lymphopenia (first change)-» thrombocytopenia—> boné marrow hypoplasia); vessel changes thrombosis (early), fibrosis (late, ischemia); skin changes including erythema—> cedema-> blistering-> chronic radiodermatitis> potential for squamous cell carcinoma; GI tract, with diarrhea Ultraviolet light (non-ionizing radiation). * UVA: Wood's lamp (black light); UVB: sunbum, comeal bums from skiing, mutagenic effect on skin (thymidine dimers), ¢ UVC: germicidal, UVB-related cancers: basal cell carcinoma is MC skin cancer; squamous carcinoma (precursor is actinic/solar keratosis); malignant melanoma Laser radiation: intense localized heat that is equivalent to a third degree bum Microwave radiation: ¢ produces a skin bum, + adversely affect pacemaker devices, « inconclusive complications: cataracts, cancer, sterility Infrared radiation: « burns, « cataracts High altitude injury: + general: O, concentration is 21% (normal); decreased barometric pressure (e.g., only 200 mm Hg on top of Mt. Everest); hyperventilation is essential at high altitude since it lowers alveolar COz (respiratory alkalosis) —> automatically increases alveolar PAOp-> increases arterial PO2, # acute mountain sickness: occurs within the first 24~36 hrs of an ascent above 8000~10,000 feet; headache, lethargy, insomnia, dyspnea; Rx with descent, increase fluid intake (increased insensible water loss from mucous membrane evaporation), and oxygen; prevention by acctimatizing before ascending and using acetazolamide, which is a carbonic anhydrase inhibitor that produces metabolic acidosis» compensation for the expected respiratory alkalosis, « high altitude pulmonary edema: non-cardiogenic Sites of alcohol (ethyl alcohol, ethanol) reabsorption: « small intestine- 75%, « stomach- * 25%, * partially metabolized by alcohol dehydrogenase Liver metabolism of alcobal: alcohol dehydrogenase aldehyde dehydrogenase (blocked by disulfiram) alcohol ————p> acetaldehyde + NADH ———} acetate + NADH ——> acetyl CoA. Pharmacologic action of alcohol: « CNS depressant in descending order» cerebral cortex-> limbic system-> cerebellum» lower brain stem, « potentiates inhibitory neurotransmitters like y- aminobenzoic acid, + wldetyde dehydrogenase deficiency: affects ~40% of Asians; build-up of acetaldehyde and GI upset (similar to action of disolfirem), « alcohol toxicity: 50 mg/dL. euphoria, ‘gregarious; 100 mg/dL legally drunk in most states (slurred specch, uncoordinated); 300~350 mg/dL stupor or coma; >500 mg/dL death, delirium tremens: following occur 3-5 days after complete withdrawal-> tremulousness, disorientation, visual hallucinations, agitation; Rx with IV diazepam and thiamine Diseases where alcohol is the leading cause; * thiamine deficiency: Wetnicke’s syndrome, Korsakoff"s psychosis, congestive cardiomyopathy, * macrocytic anemia: folate deficiency, « acquited sideroblastic anemia microcytic anemia with ringed sideroblasts, * Mallory Weiss syndrome: tear of the distal esophagus/proximal stomach from etching, * Boerhaave’s syndrome: rupture of the distal esophagus/proximal stomach from retching, © cirrhosis, © esophageal varices: effect of portal vein hypertension due to alcoholic cirrhosis, « fatty change in the liver, * hemosiderosis: alcohol increases the reabsorption of iron, * acute and chronic pancreatitis, © type IV hyperlipidemia: alcohol increases synthesis of VLDL, * Klebsiella pneumoniae pneumonia Alcohol as a cancer risk: squamous carcinoma (synergistic with smoking): oropharynx, mid- esophagus, larynx, # adenocarcinoma: pancreas, liver Alcohol effects on CNS/PNS: © Wemicke's syndrome/Korsakoff's psychosis, « cerebellar degeneration: Hu and Yo antibodies noted in spinal fluid, ¢ dementia, « DTs, « distal peripheral 4Note: This material is copyrighted, All rights reserved. Edward Goljan, M-D. 2003 neuropathy, © central pontine myelinolysis: demyelination syndrome due to rapid IV Rx of hyponatremia © Smoking epidemiology: + MCC of premature death in the United States, « MC single preventable cause of cancer, * incidence of smoking is increasing in women and decreasing in ‘men, # nicotine intake can be monitored by measuring plasma or urine Jevel of cotinine: cotinine is ‘only derived from the metabolism of nicotine, * MOA of nicotine: absorbed rapidly into the pulmonary circulation: moves into the brain where it attaches to nicotinic cholinergic receptors to produce its gratifying effects/complication of smoking; highly addictive agent, « inactivation of the ps53 suppressor gene by a point mutation on chromosome 17 is the MC genetic defect in smoking-induced cancer = Cancers where smoking is the leading cause: + lung cancer: squamous, small cell, and adenocarcinoma to a lesser extent, MCC of death due te cancer in both men and women, » oral pharyngeal and laryngeal and zid-esophageal squamous cancer, « pancreatic adenocarcinoma, « transitional cell carcinoma of bladder, # renal adenocarcinoma, © — Smoking effects on the eardiovascular/CNS systems: + increases risk for AMI: increased risk for recurrent AMI as well, » increases risk for sudden cardiac death, « increases risk for peripheral vascular disease, # increases risk for strokes, « contributing factors: enhanced atherosclerosis duc to chemicals in smoke and nicotine effect on blood pressure and heart rate; atherogenic lipid profi tissue hypoxia secondary to excess carbon monoxide (CO); hypercoagulability = Smoking effects on the respiratory system: + COPD; ~80% of all cases, chronic bronchitis, ‘emphysema, ¢ recurrent infections: pneumonia, URIs, ¢ exacerbates bronchial asthma, « cancer © Smoking effects on the GI system: * GERD, « delays the rate of ulcer healing, « increased risk for ‘oral, upper and lower GI cancer = Effects of smokeless tobacco (snuff, chewing tobacco): * nicotine addiction, oral leukoplakia/cancer: inside the lip, under the tongue or cheek, * verrucous squamous cancer, « nasal cancer— snuff users, + aggravation of cardiovascular disease: nicotine effect ‘% — Smoking effects on bone and menopause: « increases the risk for osteoporosis in men and ‘women, « biochemical reaction in women: estradiol (most potent estrogen) is normally metabolized in the liver into estrone, which is metabolized into methoxyestrone (no hormonal activity) or estrio! (trong estrogen activity)-> smokers have greater conversion of estrone into the inactive metabolite leading to low estriol levels-> low levels of estrogen increase the risk for osteoporosis and premature menopause = Passive smoking effect on children: * pathogenesis of passive smoke effects: ~ 75% of total combustion product in a cigarette is exhaled; risk of passive smoke extends to children as well as adults, « increases the incidence of SIDS, « increases risk for lung cancer: 1-2 times increased cancer risk, ¢ exacerbates asthma, « increases risk for otitis media, « increases risk for recurrent ‘upper and lower respiratory infections % — Miscellaneous smoking effects: « increases risk for developing proteinuria in diabetes mellitus, directly responsible for ~25% of residential fires, « vitamin C deficiency = Beneficial effects smoking cessation: « longevity: smokers who quit before 50 yrs of age have half the risk of dying over the next 15 yrs than a smoker has, # lung cancer: in 10 yrs, there is a 50% reduction in lung cancer when compared to a smoker, after 15 yrs, there is only a 16% risk for tung cancer when compared to a smoker, AMI: AMI risk approaches that of a nonsmoker after 1 yr of abstinence, ¢ pregnancy: pregnant women who stop smoking in the first trimester reduce the risk of a low birthweight aby ¢o that of a nonsmoker, « forced expiratory volume in 1 second (FEV ue): it isnot improved by cessation of smoking; rate of decline is similar to that of a non-smoker 1sa4 material is copyrighted. All rights reserved. Edward Goljan, M.D. 2003 Drugs and interstitial pulmonary fibrosis: + amiodarone, * bleomycin, © busulfan, © cyclophosphamide, « nitrofurantoi, » nitrosourea, « methysergide— also retroperitoneal fibrosis and Raynaud phenomenon, + methotrexate, * procarbazine Occupation exposure relationships: * automobile mechanic: carbon monoxide, * pesticide industry: organophosphates; arsenic, * meat packing: polyvinyl chloride with risk of hepatic angjosarcoma, * insulation/demolitiowroofing material: asbestos: lung cancer, mesothelioma, fibrous pleural plaques (MC overall complication of asbestos); formaldehyde, * dry cleaning: carbon tetrachloride with liver necrosis due to free radicals, + rubber/chemical industry: benzene: aplastic anemia, leukemia; aniline dyes: bladder cancer, © battery, smelter, plumber/foundry:lead poisoning; painter: methylene chloride (converted into carbon monoxide), solvents, lead, * petroleum: benzene, polycyclic hydrocarbons: lung cancer, * sewer worker: Inydrogen sulfide gas (sulfhemoglobiriemia) Isopropyl alcohol (rubbing alcohol) poisoning: + metabolism: metabolic end-product in the liver is acetone: 170 metabolic acidosis unlike other alcohols; increases serum osmolal gap: difference between calculated and measured serum osmolality >10, + clinical:deep coma with hyporeflexia Methyl alcohol (Wood's alcohol): * increased anion gap metabolic acidosis (converted into formic acid), « optic neuritis and potential for blindness, « Rx with IV ethanol: ethanol is a ‘competitive antagonist with methy\ aleahol for alcohol dehydrogenase Ethylene glycol (antifreeze): « increased anion gap metabolic acidasis (converted into oxalic acid), ‘renal failure from calcium oxalate crystals obstructing the lumens, # Rx with IV ethanol: ethanol isa competitive antagonist with methyl alcohol for alcohol dehydrogenase ‘Mercury poisoning: * MOA: toxic in inorganic (elemental) form: dental amalgams, used to be used in hat malking industry ("mad hatter disease"); toxic in organic form: fungicides, contaminated fish, « clinical: diarthea; visible on x-rays; nephrotoxic ATN involving the proximal tubules; cerebral/cerebellar neuron loss; constricted visual fields, « Rx: dimercapral Arsenic poisoning: « sources: pesticides, animal dips, Fowler’s solution; Rx of syphilis in 1930's, ‘clinical: garlic odor to breath; severe diarrhea: "rice water” stools similar to cholera; arsenic smelanosis: gray skin with dark macules; squamous cell carcinoma of skin and lungs; nails have transverse bands (Mees nails): concentrates in keratin/hair/nails; convulsions/eoma: MC COD; nephrotoxic acute tubular necrosis involving proximal tubules; liver angiosarcoma, * Rx: imercaprol ‘Mushroom poisoning (Amanita): * MOA: toxin inhibits RNA polymerase, ¢ abdominal pain/vomiting, « bloody diarthea, « jaundice: extensive fatty change Petroleum product (gasoline, kerosene) disorders: » euphoria (drunk acting) when inhaled (or ingested), « addicting, « toxic doses: convulsions, tinnitus, non-cardiogenic pulmonary edema Poisonous snake envenomations: * types: pit vipers: rattlesnakes (MC bite), wafer moccasins, copperheads; true cobras: coral snake (neurotoxin that blocks acetylcholine release paralysis and death, similar to botulinum poison); coral snake has following color banding: “red and yellow Killa fellow" (red and yellow bands next to each other), harmless scarlet king snake: "red and black friend of jack" (red and black bands next to each other), * pit viper envenomations: local swelling/necrosis;hematologic problems: DIC; antivenin is available: danger of serum sickness Testicles in hernia sac versus hydrocele: « testicles are not translucent, « translucency in the scrotal sac implies a hydrocele isual field defect with craniopharyngioma: bitemporal hemianopsia (impinges on optic chiasm) Patient with normal PT, PTT, bleeds after surgery even after receiving fresh frozen plasma and packed RBCs: quantitative platelet problem (thrombocytopenia) or qualitative. problem (patient on NSAIDs) 16Note: This material is copyrighted. All rights reserved. Edward Goljan, M.D. 2003 = Differential diagnosis of cavitary lesions in upper lobe: « MCC is reactivation TB (not primary), ‘histoplasmosis, « Klebsiella pneumoniae, « squamous cercinoma = Patient with Marfan's syndrome has chest pain radiating into the back and a diastolic ‘murmur: aortic regurgitation duc to stretching of aortic valve ring by the dissection © First step in work-up of a peripheral blood smear of microcytic anemia: order serum ferit « decreased in iron deficiency, increased in ACD, normal in mild o and B-thalassemia & Patient with twitching of face after tapping of facial nerve: « tetany due to decrease in ionized caleium- threshold potential is lowered so muscle/nerve are partially depolarized, « Chvostek’s sign, ¢ Trousseau's sign is when thumb adducts into palm when taking blood pressure = Patient taking penicillin for streptococcal infection develops hemolytic anemia with jaundice and a positive direct Coomb's: due to autoimmune hemolytic anemia (type II) against BPO attached to RBC membrane & — Main site for acetaminophen toxicity, shock, CO poisoning, alcohol in liver: around central vein least amount of oxygen in this site (zone II) % Differentiating mumps from pancreatitis: « mumps only has increase in serum amylase, * pancreatitis has inerease in amylase and lipase & Differential diagnosis of a flank mass in a child: ¢ Wilm's tumor-associated with aniridia and hemihypertrophy in the AD type of Wilms, « renal dysplasia. MC cystic disease in children = Patient with viral myocarditis develops hypotension, neck vein distention, a drop in blood pressure on inspiration, and muffled heart sounds: pericardial effusion from coxsackie myocarditis~ Bx of heart would show a lymphocytic infiltrate with destruction of muscle © Newborn baby girl, swelling of dorsum of hands/feet (lymphedema), cystic mass in neck: lymphedema in child with Turner's syndrome, # XO, « defects in lymphatics, « cystic mass in neck due to dilated lymphaties that stretch skin and produce webbed neck © Smoker with painless jaundice with clay colored stools and varicosities in right upper shoulder area: » migratory thrombophlebitis in a patient with carcinoma of the head of pancreas with obstruction of bile flow, « called Trousseau's sign = Patient with diarrhea and episodic flushing of the skin, nodular liver (metastasis from carcinoid tumor of terminal ileum), elevated urinary 5-hydroxyindoleacetic acid level, thick, plaque-like deposits found on the tricuspid and pulmonic valve leaflets: carcinoid heart disease ‘= — 16-yr-old died suddenly while playing basketball: + hyperttophic cardiomyopathy, « asymmetric hypertrophy of the interventricular septum with conduction defects, « murmur intensity increases (worse) with factors decreasing preload (standing, Valsalva, venodilator), # murmur intensity decreases (better) with factors increasing preload (sustained hand grip, squatting, lying down, B- blockers) = — Hypertensive infant has masses in the skin and lytic lesions in bone; small, basophilic staining cells $100 antigen positive, primary lesion in : adrenal medulla—neuroblastoma = Young girl with necrotic, bloody, grape-like mass protruding from the vagina; biopsy reveals malignant cells with cross-striations; tumor stains negative for cytokeratiz, factor VII related antigen, and carcinoembryonic antigen, and positive fer desmin; cancer is most likely derived from: * muscle: embryonal rhabdomyosarcoma, MC sarcoma in children © Elderly woman with abdominal distention, primary cancer most likely responsible for Induration in pouch of Douglas on rectal examination: ovarian cancer— induration due to seeding by the cancer ‘© Elderly man with point tenderness in the lower vertebral column, elevated serum alkaline phosphatase. What tests or procedures would be your first step in the evaluation of this patient: « digital rectal exam: think cheap, © bone tendemess implies stage IV disease and DRE should be positive, « increased alkaline phosphatase suggests asteobiastic bone metastasis, "7‘Note: This material is copyrighted, All rights reserved. Edward Goljan, M.D. 2003, 94 ‘Metastatic adewocarcinoma in left supraclavicular node would most likely have its primary ‘origin in: stomach adenocarcinoma with mets to Virchow/s node 15-yr old girl develops a sudden onset of right lower abdontinal pain, Pregnancy test negative. Cystic mass with densities in the right ovary noted on x-ray: cystic teratoma An ulcerated lesion that develops in a keloid secondary to a third degree burn or an ulcer located at the orifice of a chronically draining sinus that does not respond to medical management is most likely due to: development of a squamous cell carcinoma ‘Hematuria in a 58-yr old smoker would most likely be associated with which of the following ‘groups of eancers: renal adenocarcinoma/transitional cell carcinoma of the bladder both of which are most commonly caused by smoking Pneumoconiosis whose most common lesion is a benign pleural plaque: asbestosis. ‘A woman with chronic headaches develops episodic asthmatic attacks. The most likely cause is ingestion of non-steroidal anti-inflammatory agents with release of leukotrienes: classic triad asthma, # nasal polyps may also be present Medical student with intermittent bouts of diarrhea and constipation associated with cramping right and left lower quadrant pain: « irritable bowel syndrome: intrinsic motility defect in bowel, «flexible sigmoidoscopy is negative Immunizations given at birth to a baby whose mother is positive for HBsAg: « Hep B vaccine: active immunization, » HBJGa: passive immunization ‘A child has eaten raw hamburgers and now has a hemolytic anemia and renal failu due to 0157: H7 serotype of E.coli, « low platelet count and prolonged bleeding time ‘A 28-yr-old woman has an anterior mediastinal mass and non-tender lymphadenopathy in the right supraclavicular node: nodular sclerosing Hodgkin's disease ‘A 75-year-old Woman in a nursing home has non-palpable ecchymoses limited to the back of her hands. A CBC is unremarkable. The patient most likely has: senile purpura ‘Which of the following vitamin toxicities predisposes a patient who is taking warfarin to bleed and have an INR outside the normal range: vitamin E toxicity decreases the synthesis of vitamin K dependent factors in the liver A 30-year-old man with AIDS has pltting edema, hypertension, proteinuria > 3.5 /24 brs, and fatty casts in the urine, A renal biopsy exhibits glomerular disease: focal segmental ¢glomerulosclerosis, ‘A 29-year-old woman develops generalized pitting edenta and hypertension. A renal biopsy reveals diffuse glomerular disease with increased thickness of basement membranes and hypercellularity. Silver stains reveal "tram track" splitting of the basement membranes. An electron micrograph exhibits "dense deposits" in the glomerular basement membranes, The patient has a persistently low serum C3: * type II MPGN or "dense deposit disease", « patients have C3 nephritic factor: autoantibody against C3 convertase in alternative system that causes it to continually activate C3 causing very low levels ‘A 25-year-old man, who initially presented to the hospital with hemoptysis, has progressed Into renal failure. A renal biopsy shows linear lmmunofluorescence.: Goodpasture's syndrome: patient has rapidly progressive crescentic glomerulonephritis A74-year-old man with colon cancer develops generalized anasarca. He has proteinuria >3.5 2/24 ir and fatty casts in his urine. A renal blopsy shows diffuse glomerular disease. Silver stains demonstrate epimembranows spikes and the presence of subepithelial deposits on electron microscopy: diffuse membranous GN A 10-yr-old boy living In Salt Lake City, Utaht has bilateral sensorineural hearing loss, ocular abnormalities, and glomerulonephritis. Both JF and EM are negative. Foam cells are noted in ‘visceral epithelial cells: Alport's syndrome: sex-tinked dominant disease HUS 18Note: This material is copyrighted. All rights reserved. Edward Goljan, M.D. 2003, & A 65-yr-old man with chronic HCV hepatitis has a history of cyanosis of his hands, tip of nose, and ears during cold weather. These findings subside when he comes indoors. He now ihas palpable purpura in cold-exposed areas: cryoglobulinemia = A I2year-old has a history of cellulitis ~2 weeks ago. He now presents with periorbital edema, hypertension, and smoky-colored urine. Urinalysis shows RBC casts with mild proteinuria (1.5 g/24 h). Renal biopsy reveals a diffuse increase in cellularity along with a neutrophilic infiltrate. IF shows granular deposits. Subepithelial deposits are noted on EM: acute post-streptococeal GN: immunologic reaction secondary to either a group A streptococcus pharyngitis or skin infection = A 62-year-old woman with a long-history of severe rheumatoid arthritis and restrictive ‘cardiomyopathy now presents with pitting edema 2nd hypercholesterolemia. Urinalysis shows oval fat bodies and fatty casts, Protein dipstick is 2+, sulfosalicylic acid 2+. Renal biopsy shows hyalinization of the glomerular mesangium. A special stain is ordered: systemic amyloidosis = An 89-year-old woman with a history of chronic HCV hepatitis from a blood transfusion now preseats with generalized puffiness and mild hypertension. Urinalysis shows oval fat bodies, fatty casts with Maltese crosses, and 4+ protein (> 3.5 g protein/24 hr). A renal biopsy reveals hypercellular glomeruli with an increase in basement membrane thickness and tram-track splitting of the basement membranes. The IF is granular and EM reveals subendothelial deposits. The patient has depressed levels of C3: type I MPGN with HCV association A 24-year-old man with a prior history of an upper respiratory infection is noted to have microscopic hematuria and mild proteinuria (1.5 g/24 hrs) during a routine physical exam. He is not hypertensive. A renal biopsy shows a granular IF with predominantly IgA deposition in the mesangium and electron dense deposits in the same area: IgA glomenulonephritis = A 48-year-old woman has proteinaria (> 3.5 gm/24 hrs), hypertension, and fatty casts in the urine. A renal biopsy reveals eosiaophilic nodular masses in the mesangium of the glomeruli, hyaline arteriolosclerosis of the afferent and efferent arterioles, and increased thickness of the basement membranes of the tubules, IF is negative. EM reveals fusion of the podocytes and Increased collagen deposition in the mesangium and basement membranes: nodular sglomerulosclerosis in diabetes mellitus = A 28-yr-old man, with a history of renal disease in his maternal grandfather, has end-stage renal disease requiring dialysis. Physical exam reveals papular red lesions on his skin, peripheral neuropathy, and corneal disease. Previous renal biopsies exhibited vacuolated visceral epithelial cells which correspond with lamellar bodies on EM: Fabry’s disease: SXR discase with deficiency of a-galactocerebrosidase A and accumulation of ceramide trihexoside = A normotensive 10-yr-old boy, with a previous history of an upper respiratory infection, Presents with generalized pitting edema. He has had problems with allergies since early childhood. Urinalysis reveals proteinuria (> 3.5 g/24 irs), fatty casts, and oval fat bodies. His clinical findings improve rapidly with high dose corticosteroids: lipoid nephrosis ‘© A 25:yr-old woman presents with hypertension. A urinalysis reveals mild proteinuria, hematuria, and RBC casts. A serum ANA exhibits a rim pattern. An anti-dsDNA titer is extremely high: type IV SLE glomerulonephritis, © Patient with Wegener's granulomatosis develops hematuria and an abnormal urine cytology: « transitional cell carcinoma, « patient is on cyclophosphamide, which produces hemorthagic cystitis and TCC = An Egyptian man has microscopic hematuria and an abnormal urine cytology: squamous cell carcinoma: patient has Schistosoma hematobium involving the bladder plexus = An uncireumeised male has an ulcerative lesion on the undersurface of the glans penis and palpable inguinal adenopathy: © squamous cell carcinoma (penis), « lack of circumcision is ‘greatest risk factor 19Note: This mater is copyrighted. All rights reserved. Edward Goljan, M.D. 2003 © Aman, who has worked with aniline dyes in the past, develops microscopic hematuria and an abnormal urine cytology: « transitional cell carcinoma, * most likely it is bladder, but it could be renal pelvis as well © A Tiyr-old man has bilateral painless masses in his testicles: © malignant lymphoma: metastatic, not primary, « note the bilaterality = A patient with acute myelogenous leukemia developed acute renal failure after being treated ‘with multiple chemotherapy agents: increased release of purines led to increased production of uric acid, whose crystals blocked the renal tubules resulting in ARF = Complication associated with sickle cell trait, acute pyelonephritis, analgesic abuse, and diabetes mellitus: renal papillary necrosis = Patient who works outdoors all the time rubs a gray-white lesion off from the back of his hand but it grew back after a few weeks: actinic (solar) keratosis- precursor for squamous cell carcinoma © — Inborn error of metabolism associated with pellagra: Hartnup's disease: loss of neutral amino acids in the GI and GU tract; loss of tryptophan leads to pellagra due to decrease in synthesis of @ African American took primaquine and then developed an anemia with bite cells: G6PD deficiency = Patient with intense occipital headache and blood in CSF: subarachnoid bleed in a patient with a ruptured congenital berry aneurysm © Sexually active man with sterile pyuria and no organisms seen on gram stain: Chlamydia trachomatis non-specific urethritis = Patient that just returned from a trip to Australia develops severe dyspnea and chest pain: developed DVT in the lower legs from sitting too long and now has a pulmonary embolus with infarction = — Man with severe hyponatremia and high urine osmolality: + inappropriate ADH syndrome from small cell cancer in the lung, * high urine osmolality is due to increased peritubular hydrosta pressure and loss of sodium in the urine as well as constant concentration of the urine from excess ADH Intravenous drug abuser with fever and pansystolic murmur increasing on inspiration: tricuspid regurgitation due to 5. aureus infective endocarditis Down syndrome patient with vomiting and maternal polyhydramnios: duodenal atresia Down syndrome patient with constipation at birth: Hirschsprung’s disease Complications of prostate hyperplasia: + infection, + urinary retention, « bladder diverticula Patient with a mass in falloplan tube and an increase in B-hCG: ectopic ‘A patient in the hospital accidentally gets infused with saline contaminated with E. coli and develops shock. Two days later there is bleeding from all puncture sites. Diagnosis? « DIC, + best test is D-dimers ‘MC vitamin deficiency in alcoholics: folate ‘Woman is a pure vegan and is breast feeding her baby and the baby develops anemia: Bi2 deficiency © — When to give folate to prevent open neural tube defects: * before pregnancy, already developed before the patient knows she is pregnant Interpretation of a Schilling's test corrected with IF: PA. Drug causing a macrocytic anemia in a patient with severe rheumatoid arthriti which blocks dihydrofolate reductase © Drug young woman with hypertension and a macrocytic anemia is taking: « birth control pills, ‘estrogen increases angiotensinogen, « estrogen blocks reabsorption of monoglutamate a aggag aa neural tube is a4 : methotrexate, 20Note: This materi copyrighted. All rights reserved, Edward Goljan, M.D. 2003, © Anemia in an infant that develops when switched from cow's milk to goat's milk: # goat's milk is low in folate, Bg (pyridoxine), iron and high in potassium, chloride, arachidonic acids, and linoleic acids when compared to cow's milk = Prevention of macrocytic anemia in patients on methotrexate: « leucovorin rescue (citrovorum factor), « replenishes N“"°-methylene THF: substrate necessary for thymidylate synthesis & An AIDs patient develops diarrhea with an acid-fast organism. What is the most likely cause?: Cryptosporidium © A patient with liver cirrhosis and ascites develops spontaneous bacterial peritonitis, what is ‘the most likely organism?: E. coli © Newborn has a puffy face, poor muscle tone, and diminished deep tendon reflexes: cretin (hypothyroidism) = Previously healthy gir? develops aplastic anemia, Cause?: © — Mother with polyhydramnios and fetus with anal atresia: VATER syndrome~ vertebral defects, anal atresia, fracheoesophageal fistula, radial (absent) and renal abnormalities © Newborn with tetany and small ears: « DiGeorge syndrome, » 3rd and 4th pharyngeal pouches do not develop (absent thymus and thymic shadow and absent parathyroids), « expect hypocaleemis = Women with excessive welght loss (e,, marathon runner, aerobies instructor, anorexia nervosa): decrease secretion of GnRH from hypothalamus leading to decrease in gonadotropins-» secondary amenorrhea» osteoporosis (lack of estrogen) = — Post-radical mastectomy—winged scapula: indicates injury to the long thoracle nerve; paralysis of the serratus anterior muscle & Paralysis of upward gaze in an infant: hydrocephalus secondary to stenosis of the aqueduct of Sylvius, « this is called Parinaud’s syndrome © — Respiratory bronchiole: « last airway structure with cilia, « respiratory unit where gas exchange ‘occurs is the respiratory bronchiole, alveolar duct, and alveoli © Terminal bronchioles: « cilia but no goblet cells, « site of obstruction in asthma, cystic fibrosis, and chronie bronchitis, « site where turbulent air flow becomes laminar due to parallel branching of the airways = — Cogonia at birth: » arrested in prophase I of meiosis and are called primary oocytes, # arrested cells are diploid (2n) and have 46 chromosomes = Primary oocyte: « primary oocyte completes the first meiotic division within the mature follicle shortly before ovulation-> secondary oocyte, * secondary oocyte is haploid (In) and contains 23 chromosomes, + it enters the second mefotic division but is arrested in metaphase II at the time of ovulation, + the secondary oocyte completes meiosis II during fertilization © — Rotator cuff tear: » common cause of shoulder pain, * components of the rotator cuff include the tendon insertions of: supraspinatus, infraspinatus, teres minor, subscapularis muscles, * S/S of rotator cuff injuries: pain/weakness with active shoulder abduction = Tennis elbow: « pain occurs in the area where the extensor muscle tendons insert near the lateral epicondyle, « common in: raquette sports, repetitive use of a hammer or screwdriver Golfer's elbow: pain is located where the flexor muscle tendons insert near the medial epicondyle Ulnar nerve compression: « the ulnar nerve may be entrapped in the following areas: transverse carpal ligament, elbow ("funny borte area"), there is pain and numbness of the ulnar aspect of the forearm and ring and little finger plus weakness of the intrinsic muscles of the hand © — Compartment syndromes: + increase of pressure in a confined space: pressure reduces perfusion, ‘which may lead to permanent ischemic contractures of the muscle (s) in that compartment, « S/S of compartment syndrome: pain, paresthesias, pallor, paralysis, pulselessness, * causes of compartment syndromes: fractures, injuries to arteries/soft tissue, prolonged limb compression (ca, tight fitting cast), ¢ Volkmann's ischemic contracture: complication of a supracondylar fracture of the humerus; there is injury to the brachial artery and median nerve; brachial artery a4 aNote: This material is copyrighted. All rights reserved. Edward Goljan, M.D. 2003 ischemia leads to increased pressure in the closed muscle compartments of the forearm with a subsequent decrease in venous and then arterial perfusion: this may lead to permanent ischemic contractures of the muscle Carpal tunnel syndrome: » entrapment syndrome of the median nerve in the transverse carpal ligament of the wrist, * causes: rheumatoid arthrtis/pregnancy, overuse of the hands/wrist, amyloidosis, hypothyroidism, * S/S: pain, numbness, or paresthesias in the thumb, index finger, second finger, third finger, and the radial side of the fourth finger; thenar atrophy ("ape hand appearance); pain is reproduced by tapping over the median nerve "Claw hand": © ulnar nerve palsy, © ulnar nerve functions: adduction of the fingers due to innervation of the palmar and dorsal interosscus muscles; adduction of the thumb (adductor pollicis muscle) "Wrist drop": « radial nerve plsy, + radial nerve functions: extensor muscles of the wrist and digits; wrist drop refers to a hand that is flexed at the wrist and cannot be extended, « injury may be due to: midshaft fractures of the humerus; draping the arm over a park bench (called "Saturday night palsy” "Walter's tp deformity": + brachial plexus lesion involving C; and Ce: upper trunk injury, « clinical findings in Erb-Duchenne's palsy: birth injury of the brachial plexus with damage to Cs-C6 nerves upper trunk injury; loss of abduction of the arm from the shoulder; inability to externally rotate the arm; inability to supinate the forearm; absent biceps reflex; asymmetric Moro reflex: no movement on the affected side Klumpke's paralysis: « birth injury of the brachial plexus with injury to the C;-C, and T,- lower trunk injury, * paralysis of the hand, « Homer's syndrome Axillary nerve injury (C,-C.): « fracture of the surgical neck of the humerus, # dislocation of the shoulder joint: usually anterior dislocation; may also injury the axillary artery, ¢ patient cannot abduct the arm to the horizontal position of hold the horizontal position when a downward force is applied to the arm: paralysis of deltoid muscle, weakening of lateral rotation of the armm:paralysis of teres minot muscle Herniation of Ls-S, aise : » sensory loss: lateral and posterior calf; plantar aspect of the foot, « reflex loss: Achilles reflex (tibial nerve L,-S;), © motor deficit: loss of plantar flexion, loss of foot eversion (weakness of peroneus longus/brevis) Herniation of L.-Ls dise: * sensory loss: dorsum of the foot, webbed space between the great toe, « reflex loss~ none, « motor deficit: loss of dorsiflexion of the big (great) toe due to weakness of the extensor halluels longus Herniation of Ls-Ls disc: * sensory loss: medial leg to the malleolus, + reflex loss: knee jerk. (femoral nerve L-L1), # motor deficit: quadriceps weakness due to weakness of knee extension; loss of dorsiflexion of the foot due to weakness of the tibialis anterior Superior gluteal nerve injury: + causes: surgery; Duchenne’s muscular dystrophy, * waddling ‘galt: the superior gluteal nerve supplies the gluteus medius and minimus muscles, lose of abduction of the thigh and inability to pull the pelvis down; positive Trendelenburg's sign: raising of the foot on the injured side causes Peroneal nerve injury: « MC site of entrapment is behind the knee: common in people who cross their legs a lot; patient has a slapping gait, « motor deficits: loss of foot eversion due to weakening. of the peroneus longus and brevis muscles; loss of foot dorsiflexion due to weakening of the tibialis anterior muscle; loss of toe extension due to weakening of the extensor digitorum longus and hallucis longus muscles; combined effect of all the above produces an equinovarus deformity (plantar flexion with foot drop and inversion of the foot), « loss of the ankle jerk reflex Kyphosis in adolescents: » forward bending of the spine, « MCC is muscular/postural problems: commonly seen in tall adolescents 22‘Note: This material is copyrighted. All rights reserved. Edward Goljan, M.D. 2003 g q sett al a4 Scoliosis: * term applied to lateral curvature of the spine, « idiopathic scoliosis: usually affects adolescent girls between 10-16 yrs of age; usually a right thoracic curve; forward bending causes a ace on the right from a hump in the ribs due to a rotational component of the ‘Test used to evaluate meniscus injuries: McMurray test: test for the medial meniscus: patient is supine-> knee is acutely flexed foot is grasped and the leg is externally rotated-> knee is slowly extended while the other hand feels the posteromedial margin of the knee joint» a click along the posteromedial margin indicates a medial meniscus tear: femur passes over the tear, + test for a lateral meniscus injury: same procedure as above except the leg is rotated internally and extended» a clickis palpated along the posterolateral margin of the joint Location and function of the anterior cruciate ligament (ACL): ACL attaches the anterior part of the tibia to the lateral condyle of the femur— prevents anterior movement of the tibia in relation to the femur Location and function of the posterior cruciate ligament (PCL): PCL extends from the posterior part of the tibia to the medial condyle of the femur~ prevents posterior movement of the tibia in relation to the femur Tests used to evaluate the cruciate ligaments in the knee: « anterior draw test to evaluate ACL: patient supine, hip flexed 43°, knee flexed 90°-> examiner places hand on the posterior aspect of the tibia—> anterior force is applied in neutral, extemal, internal direction» positive anterior draw test is when there is anterior displacement of the tibia, « posterior draw test to evaluate PCL: patient supine, hip flexed 45°, knee flexed 90° examiner places fand on the anterior aspect of proximal tibia» posterior force is applied in neutral, external, intemal direction» positive posterior draw testis when there is posterior displacement of the tibia Positions the knee joint is forced into that results in injury: + valgus position: angulation away from the midline, e.g clipping injury in footbail,« varus position: angulation towards the midline, medially originating force is applied to the knee SIS of a meniscus injury: * pain, © knee catches, locks, or gives way when walking, © sswelling/popping of the knee Medial meniscus injury: « mechanism of injury: MC internal derangement of knee joint; most commonly part of 4 valgus injury (e.g., clipping in football), * structures damaged: medial ‘meniscus; medial collateral ligament; ACL, positive McMurray’s test: click on posteromedial margin with the knee flexed, extemally rotated, and slowly extended ACL injury: + mechanism of injury: MC ligament injury; tom most commonly in a valgus injury secondary to clipping or skiing, # positive anterior draw sign PCL injury: « mechanism of injury: hyperextension of the knee secondary to an anterior force pushing the tibia in a posterior direction, « positive posterior draw test Medial collateral ligament injury: tom with valgus Lateral collateral ligament injury: tom with varus injuries ‘MC nerve injured with clavicular fractures: ulnar nerve MC fracture associated with falling on the outstretched hand: « Colles fracture of the distal radius: MC fracture of the wrist; “dinner fork” deformity of the proximal radial fragment (displaced upward and backward); second most common fracture in osteoporosis in women MC femoral fracture: * femoral neck fracture: most commonly occurs in the elderly male patient with osteoporosis; groin or knee pain is present, « complications: aseptic necrosis of femoral head due to damage to the medial femoral circumflex artery ‘MC foot bone fractured after a fall from a height: calcaneus PCR mechanism: uses DNA polymerase to break down DNA into fragments 23‘Note: This materlal Is copyrighted. All rights reserved, Edward Goljan, M.D. 2003, © Nutrient and Energy Requirements in Humans ‘A. The normal diet provides fuel for energy, precursors from which important body components are derived (e.g., essential fatty acids and stino acids), micronutrients (eg, vitamins, minerals, trace metals), and water. B. Dietary requirements for protein, vitamins, and minerals and trace elements are specified in terms of the recommended daily allowance (RDA). 1. RDA represents an optimal dietary intake of nutrients that under ordinary conditions will keep the general population in good health. 2. RDA varies with sex, age, body weight, diet, and physiological status. + Forexample, RDAs for nutrients increase in childhood, pregnancy, and lactation. C. Daily energy expenditure depends on the basal metabolic rate (BMR), thermic effect of foods, and the degree of physical activity. 1. BMR accounts for ~60% of daily energy expenditure and refers to the energy consumption of an individual at rest. a. BMR reflects the energy involved in normal body functions (¢.g., cardiac function, maintaining ion pumps) and primarily depends on body weight. © Anestimate of BMR is obtained by multiplying the body weight in kilograms (1 kg equals 2.2 Ibs) by 24: BMR = 24 x weight in kg b. Other factors affecting BMR include gender (males higher than females), fever (increases BMR), age (children higher than adults), and thyroid function (increased in hyperthyroidism and decreased in hypothyroidism). 2, Thermic effeet of foods accounts for ~10% of dally energy expenditure and represents the energy used in digestion, absorption, and distribution of nutrients. 3. Physical activity Is variable and is expressed as an activity factor, which when ‘multiplied by the BMR equals the daily energy expenditure. ‘©The activity factor for a sedentary person is 1.3, moderately active person 1.5, and very active person (¢.g., manual digging, marathon runner) 4.7. 4, Sample calculations: calculate the BMR and daily energy expenditure for a 220 Yo man (patient A) with sedentary habits and a 110 Ib woman (patient B), who runs 10 miles a day and is an aerobic exercise instructor at night. ‘a. Patient A: 220 Ib = 220/2.2 = 100 kg. BMR: 24 x 100 = 2400 keal/day. Daily energy expenditure: 2400 x 1.3 = 3120 keal/day b. Patient B: 110 Tb = 110/2.2 = 50 kg. BMR: 24 x 50 expenditure: 1200 x 1.7 = 2040 keal/day = Dietary Fuels: A. Carbohydrates, fats, and protein (lesser extent) are the primary fuels. B. Carbohydrates have a protein-sparing effect. 1. An adequate intake of carbohydrate provides the glucose that is necessary for complete oxidation into CO; and 1,0 via glycolysis and the citric acid cycle. 2. An inadequate intake of carbohydrate (e.g., < 150 g/day) causes degradation of skeletal ‘muscle with the release of amino acids that are deaminated and converted into substrates (e.g, pyruvate) for gluconcogenesi C. Protein energy malnutrition results from inadequate intake of protein and/or calories. 1, Marasmus is caused by a diet deficfent in both protein and calories (e.g, total calorie deprivation). ‘© It is marked by extreme muscle wasting ("broomstick extremities") due to breakdown of muscle protein for energy, as well as growth retardation, 2, Kwashiorkor is caused by a diet inadequate in protein in the presence of an adequate caloric intake consisting primarily of carbohydrates. 1200 keal/day. Daily energy 24Note: This material is copyrighted. All rights reserved. Edward Goljan, M.D. 2003 (1) The disease is marked by pitting edema and ascites (loss of the oncotic effect of albumin), enlarged fatty liver (decreased apolipoproteins), anemia, diarrhea (loss of brush border enzymes), and defects in cellular immunity. (2) There is less extreme muscle wasting than occurs in marasmus due to the protein- sparing effect of carbohydrates. = Dietary recommendations: ‘A. The body mass index (BMI) is primarily used to determine whether a person's body weight is in the acceptable range. 1. The BMTis the patient's weight in kg divided by the helght in meters squared: BMI weight in kg/helght m*, 2. A BMI between 20-25 kg/m? is acceptable; 25-29.9 kg/m’ is overweight; 30-39.9 kg/m? is obese; and, greater than 40 kg/m? is considered morbidly obese, B. The recommended distribution of total calorie intake in a diet is 50-60% from carbohydrates; no more than 30% from fats (ideal is <25%); and 10-20% from proteins. 1. The BMI should be in the acceptable range (e.g., 20-25 kg/m’). 2. Fat distribution should be 10% monounsaturated fats, 10% polyunsaturated fats, and 10% saturated fats. 3. Cholesterol intake should be less than 300 mg/day. 4. Protein intake should be 0.8 kg/day C. Imorder to lose welght, the total calories expended must be greater than the total intake of calories. 1. When primarily drawing upon adipose to meet energy needs, a person loses ~1 Ib whenever they expend 3,500 calories more than they consume. 2, Sample calculations using patients A and B: a. Patient A consumes 3600 kcal/day consisting of 168 g of fat, 108 g of protein, and 414 g of carbohydrates, Calculate the percentage of each of the nutrients. Is the patient gaining, maintaining, or losing weight? (0) Fat kcal: 168 gx 9 keal/g = 1512 keal/day; fat percent = 1512/3600 = 42% (too high). (2) Protein keal: 108 g x 4 keal/g = 432 keal/day; protein percent = 432/3600 = @) I: 1656/3600 = 46% (slightly decreased). (4) Since the patient's caleulated daily energy expenditure is 3120 keaV/day (see previous calculation), the patient is consuming more calories (3600 keal/day) than he is expending. A net gain of 480 kcal/day results in a gain of 1 Ib in ~7 days (3500/480 = 7.3). b. Patient B consumes 2000 keal/day consisting of 67 g of fat, 60 g of protein, and 290 g of carbohydrates. Calculate the percentage of each of the nutrients. Is the patient gaining, maintaining, or losing weight? (1) Fat keal: 67 gx 9 keal/g = 603 keal/day; fat percent = 603/2000 (2) Protein keal: 60 g x 4 keal/g = 240 keaV/day; protein percent (normal). (3) Carbohydrate keal: 290 g x 4 kcal/g = 1160 keal/day; carbohydrate percent = 1160/2000 = 58% (normal). (4) Since the patient's calculated daily energy expenditure is 2040 keal/day (see previous calculation), the patient is almost consuming (2000 kcal/day) the same number of calories she is expending and will likely maintain her weight. = Water soluble vitamins A. Water-soluble vitamins generally function as cofactors in enzyme reactions. B. They re readily excreted in the urine, and rarely reach toxic levels. 2% carbohydrate percent = }0% (normal). 40/2000 = 12% 25‘Note: This material is copyrighted. All rights reserved. Edward Goljan, M.D. 2003 C ‘Thiamine (vitamin By) 1. Thiamine is present in unrefined cereal grains, Brewer's yeast, meats, legumes, and nuts. 2, Thiamine pyrophosphate is the active form of the vitamin. 3. It is a cofactor for dehydrogenases in oxidative decarboxylation of a-keto acids (c. pyruvate dehydrogenase conversion of pyruvate into acetyl CoA) and for transketolase (two-carbon transfer reactions) in the pentose phosphate pathway. 4, ‘Thiamine deficiency most commonly occurs in alcoholics or malnourished individuals. a. The majority of the clinical findings in thiamine deficiency are due to loss of ATP from dysfunction of the pyruvate and a-ketoglutarate dehydrogenase reactions which normally gain 2 NADH (6 ATP). b. Intravenous infusion of a glucose-containing fluid may precipitate acute thiamine deficiency in alcoholics (used up by pyruvate dehydrogenase reaction) manifested by the Wemicke-Korsakoff syndrome (see Table). Riboflavin (vitamin B,) 1. _ Riboflavin is present in meats, poultry, fish, and dairy products. 2. FAD and FMN are the active forms of the vitamin. a. FAD is produced in the citric acid cycle, b. MN is a component of the electron transport chain (ETC) and accepts two hydrogen atoms (becomes FMNH;) from NADH in a reaction catalyzed by NADH dehydrogenase. 3. Riboflavin deficiency is usually secn in severely malnourished individuals or pure vegans, who lack intake of dairy products. Niacin (vitamin By or nicotinic acid) 1. Sources of niacin include meat, lime-treated (feleases bound niacin) of fortified cereals, and tryptophan-containing proteins. ‘The two active forms of niacin are NAD" and NADP’. a. Excess tryptophan is metabolized to niacin and supplies ~10% of the wiacin RDA. 'b, NAD" and NADP* are important cofactors in redox reactions. « NAD‘ reactions are primarily catabolic (c.g, glycolysis). ‘+ NADP* reactions are primarily anabolic (c.g., FA synthesis). 3, Niacin deficiency, known as pellagra, primarily occurs in people who have diets deficient in both niacin and tryptophan or in conditions where tryptophan is fost in urine and feces (eg,, Hartnup's disease) or excessively utilized (e.g, carcinoid Syndrome) Corn-based diets are particularly prone to pellagra since maize protein has a low ‘tryptophan content and niacin is in a bound form that cannot be reabsorbed. bv. Hartaup’s disease is an AR disease with a defect in the intestinal and renal reabsorption of neutral amino acids (e¢., eyptophan).. ©. In the carcinoid syndrome, tryptophan is used to synthesize serotonin, which produces the flushing and diarthea associated with the syndrome. 4. Clinical findings associated with pellagra: see Table 4, Excessive intake of niacin leads to flushing due to vasodilatation. Pantothente actd (vitamin B,) 1. Itis a component of cofactor A (CoA) and the fatty acid synthase complex, whi involved in FA synthesis. 2. Pantothenic acid deficiency is uncommon, Pyridoxine (vitamin B,) 1. _Itis present in poultry, fish, liver, pork, soy beans, and nuts. 2. Pyridoxal phosphate is the active form of the vitamin, 26‘Note: This material is copyrighted. AM rights reserved. Edward Goljan, M.D. 2003 Funetions of pyridoxine a. teansamination reactions (reversible conversion of amino acids to a-ketoacids), which are catalyzed by the transaminases alanine aminotransferase (ALT) and aspartate aminotransferase (AST). cofactor for 8-aminolevulinic acid (ALA) synthase, which catalyzes the rate- limiting reaction that converts succinyl CoA + glycine into 6-ALA in heme synthesis. synthesis of neurotransmitters such as y-aminobutyrate (GABA), serotonin, and norepinephrine. 4. cofactor in decarboxylation reactions (¢.g., conversion of histidine to histamine), glycogenoiysis (e.g., glycogen phosphorylase), deamination reactions (e.g., conversion of serine to pyruvate and ammonia), and conversion of tryptophan to niacin. Pyridoxine deficiency is most commonly seen in alcoholics and in patients receiving Isoniazid therapy for tuberculosis. * Pyridoxine deficiency is also noted in countries that use unfortified goat’s milk. H. Cobalamin (vitamin Buy; contains cobalt): in notes Ascorbie acid (vitamin C) I It is present in citrus fruits, potatoes, green and red peppers, broccoli, tomatoes, spinach, and strawberries. 2, Funetions of vitamin C a. Hyéroxylation of lysine and proline residues during collagen synthesis b. Antioxidant activity (sequesters free radicals) cc. Reduces non-heme iron (+3 valence) from plants to the ferrous (+2 valence) state for reabsorption in the duodenum 4. Keeps tetrahydrofolate (FH) in its reduced form €. Cofactor in the conversion of dopamine to norepinephrine in catecholamine synthesis 3. Causes of vitzmin C deficiency (scurvy) include diets lacking fruits and vegetables (“tea and toast” diet) and smoking cigarettes. 4, Excess intake of vitamin C may result in renal stone formation. Vitamin Clinical symptoms and signs of deficiency ‘Thiamine (B,) * Wernicke-Korsakoff syndrome (confusion, ataxia, nystagmus, ophthalmoplegia, antegrade and retrograde amnesia, precipitated by giving intravenous with glucose), « peripherai neuropathy (Gry beriberi), « congestive cardiomyopathy (wet beriberi). Riboflavin (B;) + Comeal neovasculatization, « glossitis (magenta tongue), * cheilosis (cracking of lips), « angular stomatitis (fissuring comers of mouth). Niacin B:) ‘Diarrhea, + dermatitis (hyperpigmentation in sun-exposed areas), dementia. Pantothenic acid * None described. By) Pyridoxine (B.) *_Sideroblastic anemia (microcytic. anemia with ringed sideroblasts), « peripheral neuropathy, + convulsions. Vitamin Clinical symptoms and signs of deficiency Cobalamin (Br) ¢ ~Macrocytic (megaloblastic anemia (increased mean corpuscular volume), * neutropenia and thrombocytopenia, * hypersegmented neutrophils, « glossitis, « subacute combined degeneration (demyelination in posterior columns and lateral corticospinal tract), © dementia, © achlorhydria, trophic gastritis body and fundus, increased serum gastrin (only 27Note: This material is copyrighted. All rights reserved. Edward Goljan, M.D. 2003, Folic acid iotin Ascorbic acid (vitamin C) pernicious anemia), * increased plasma homocysteine, © increased urine methylmalonic acid. '* Same as vitamin Biz deficiency with the following exceptions: no neurologic dysfunction and normal urine methylmalonie acid. «Dermatitis, «alopecia, « glossitis, « lactic acidosis. « Bleeding diathesis (ecchymoses, hemarthroses, bleeding gums, perifollicular hemorrhages), « poor wound healing, « glossitis. > Fat-Soluble Vitamins A. Functions: hormones, cofactors, hemostatic agents, and antioxidants. B. They include vitamins A, D, E, and K and are absorbed with fats, transported in chylomicrons, and stored in the liver and adipose tissue. C. Toxicity can occur. D. Vitamin A (retinol) 1. Tt is present in cod liver oil, dairy products, eggs, dark green leafy vegetables (e.g., parsley), and carrots. 2, Retinol (alcohol), retinal (aldehyde), and retinoic acid are the active forms of vitamin A. a. B-Carotenes (provitamin A) found in vegetables and retinol esters in the diet are converted into retinol, which is esterified (forming retinol esters) in the enterocytes of ‘the small intestine. + An excess of B-earotenes in the diet turns the skin yellow, but the sclera remains white (unlike jaundice). b. Retinol esters are packaged into chylomicrons and transported to the liver for storage. ¢. Retinol is reteased from the liver, complexes with retinol-binding protein (RBP), and is delivered to target tissues throughout the body (except heart and skeletal muscle). 4. In the cytosol, retinol is irreversibly oxidized to retinoic acid. ¢. Retinoic acid (similar to steroid hormones and vitamin D) binds to nuclear Feceptors forming a complex that activates gene transcription of protein products. 3. Funetions of vitamin A a. Component of the visual pigments within rod and cone cells of the retina b. Important in normal cell differentiation and prevents epithelial cells from undergoing squamous metaplasia cc. Important in normal bone and tooth development 4d. Supports spermatogenesis and placental development ¢. Used in the treatment of skin disorders and acute promyelocytic leukemia (1) Topical tretinoin (all-trans-retinoic acid) is used in the treatment of psoriasis and mild acne. 2) Oral isotretinoin is used to treat severe aystic acne, but it is teratogenic, so ‘women must have a pregnaney test before itis prescribe: @) Alltrans-retinoic acid is used to treat acute promyelocytic leukemia (M3) and is thought to induce maturation of the leukemic cells, 4, Causes of vitamin & deficiency include a diet poor in yellow and green vegetables and fat malabsorption (cg,, celiac disease). 5, Causes of vitamin A excess include eating bear liver (hunters) and isotretinoin thecapy. FE. VitaminD It's present in liver, egg yolk, saltwater fish (cod liver oil), and fortified foods. 28Note: This material is copyrighted. All rights reserved. Edward Goljan, M.D. 2003 ‘Synthesis of calcitriol (1,25-dihydroxycholecalciferut), the active form of the vitamin, ‘occurs in the following sequence. 2, Preformed vitamin D in the diet consists of cholecalciferol (vitamin D,) and ergocaleiferol (vitamin D;, found in plants), which is interconvertible with vitamin Ds b. Endogenous vitamin D is produced by photoconversion of 7-debydrocholesterol to vitamin Ds in sun-exposed skin (most important source of vitamin D). c. The first hydroxylation, which produces 2S-hydroxycholecalciferol, occurs in the liver within the cytochrome P450 system, The second hydroxylation, by 1-a-hydroxylase, produces 1,25-dihydroxy- cholecaleiferol (calcitriol) and occurs in the proximal tubules of the kidneys. ©The activity of 1-c-hydroxylase in the kidneys is enhanced by parathyroid hormone (PTH) and low serum phosphorus levels. ¢. Receptors for vitamin D are located in the intestine, kidneys, and on osteoblasts in bone. 3. Functions of vitamin D a. It inereases intestinal reabsorption of calcium and phosphorous and renal reabsorption of calcium, (1) Reabsorption of calcium and phosphorous provides an adequate solubility product (calcizat x phosphorus) for vitamin D to mineralize bone: the crystalline salt deposited in bone is called hydroxyapatite Cayo(PO.)«(OH)2 (2) When vitamin D interacts with its receptors on osteoblasts, alkaline phosphatase is released: it hydrolyzes pyrophosphate, an inhibitor of bone mineralization. b. Along with PTH, it increases the mobilization of calcium from bone by stimulating the conversion of macrophage stem cells in the bone marrow into osteoclasts. * Along with PTH, calcitriol maintains the serum calcium concentration. 4, Causes of vitamin D deficiency a. Renal failure (most common cause): due to deficiency of 1-c-hydroxylase enzyme b. Fat malabsorption: cannot reabsorb fat-soluble vitamins (e.g, celiac disease) c. Chronic liver disease: cannot carry out the frst hydroxylation of vitamin D3 4 Enhanced liver oytachrome P450 system (c.g., alcohol, phenytoin, barbiturates): increased metabolism of 25-hydroxycholecalciferol. fe, Inadequate exposure to sunlight: decreased synthesis of cholecalciferol (D:) f. Primary hypoparathyroidism: PTH is required for enhancing the activity of 1-c- hydroxylase g. Type I vitamin D-dependent rickets: deficiency of 1-a-hydroxylase h. Type Il vitamin D-dependent rickets: deficiency of vitamin D receptors in target tissue 5. Patients taking megadoses of vitamin D may develop vitamin D toxicity. F Vitamin E 1. q-Tocopherol has the highest biological activity of the naturally occurring tocopherols that constitute vitamin E and is abundant in fruits, vegetables, and grains. 2. Ibis an antioxidant and scavenger of free radicals that protects polyunsaturated fats and FAs in cell membranes from lipid peroxidation and also protects low density lipoprotein (LDL) from oxidation. * Oxidized LDL is more likely than non-oxidized LDL to cause atherosclerosis, 3. Vitamin E deficiency is uncommon and primarily occurs in children with malabsorption secondary to eystic fibrosis and in abetalipoprateinemia. 4. Patients taking megadoses of vitamin E may develop vitamin K, toxicity. 29Note: This material is copyrighted. All rights reserved, Edward Goljan, M.D. 2003 G. Vitamin k L ‘Vitamin Vitamin A Vitamin D Vitamin E Sources of vitamin K include green leafy vegetables and bacterial synthesis of inactive vitamin Ka in the colon (50% of total amount). a. After reabsorption in the bowel, vitamin K is oxidized in the microsomal enzyme by an epoxidase for storage as an inactive epoxide. b. To be catalytically active (vitamin K,), it must be reduced by the microsomal enzyme epoxide reductase. Vitamin K, y-carboxylates glutamate residues in the vitamin K-dependent coagulation factors, which are II (prothrombin), VIL, IX, X, and proteins C and S. a. The vitamin K-dependent coagulation factors are ail syathesized in the liver in a non-fanetional state. b. When y-carboxylated in the liver by vitamin K, and released into the cireulation, they have the capacity ta bind to calcium and be utilized in the formation of a clot. c. The prothrombin time (PT) is a coagulation test that evaluates all of the vitamin K-dependent factors except factor IX and proteins C and S, 4. Coumarin-derivatives act as anticoagulants by inhibiting the activity of hepatic epoxide reductase, hence the vitamin K-dependent factors are rendered non- functional by their inability to bind to calcium. * Coumarin-derivatives are present in rat poison, which is a common childhood poisoning. ‘The most common cause of vitamin K deficiency is the use of broad-spectrum antibloties, which destroy colonic bacterial synthesis of the vitamin. Other causes include— a. Therapy with coumarin-derivatives: inhibits hepatic epoxide reductase ’b. Fat malabsorption: cannot reabsotb fat-soluble vitamins (e.g, celiac disease) ©. Newborns: lack bacterial colonization of the bowel and must receive an intramuscular vitamin K injection at birth to prevent hemorrhagic disease of the newborn 4. Breast milk: inadequate supply of vitamin K Excessive intake of vitamin K teading to toxicity {s uncommon. Clinical findings associated with vitamin K deficiency and excess: see Table ‘Signs and symptoms of deficleney _Signs and symptoms of excess + Night blindness (ayctalopia), + eye © Increased intracranial pres- abnormalities (dry eyes, blindness), « sure (papilledema, skin abnormalities (follicular convulsions), # ‘wyperkeratosis, dry skin), © lung liver toxicity, « bone pain. abnormalities (bronchitis, pneu-monia, 7 Tung cances), * growth retardation, « poor wound healing, # Rickets in children and osteomalacia « Hypercalcemia, kidney in adults, « findings in both conditions: stones. path-ologic fractures, excess osteoid, bowed legs, « findings in rickets only: craniotabes (oft skull), skeletal deformities, rachitie rosary, defective epiphyseal plates with growth retardation. ‘+ Hemolytic anemia, « peripheral neuro- * Decreased synthesis of pathy, © dorsal column degeneration vitamin K-dependent (poor joint sensation, absent vibratory coagulation factors (enhances 30‘Note: This material is copyrighted, All rights reserved. Edward Goljan, M.D. 2003 Vitamin K, sensation), * retinal degeneration, © anticoagulation effect of myopathy. coumarin derivatives). # Bleeding diathesis (gastrointestinal © Hemolytic anemia and bleeding, ecchymoses), * prolonged jaundice in newboms if mother prothrombin time. receives excess vitamin K. © Minerals and electrolytes A. Minerals (RDA > 100 mg/day) and trace elements (RDA < 100 mg/day), along with vitamins, are micronutrients that are required in the normal diet. B. Most minerals are found in the body fluids as electrolyte solutions. C. Calcium 1 a Sources of calcium include dairy products, green leafy vegetables, legumes, nuts, and whole grains. Functions of calcium, 1, Bone formation and teeth b. Nerve conduction c. Skeletal, cardiac, smooth muscle contraction d. Binds to vitamin K-dependent coagulation factors and activates factor XIII to cross- link fibrin strands fe. Calcium-calmodulin complex activates enzymes for signal transmission across membranes (e.g., adenylate and guanylate cyclase, phosphodiesterase) Regulation of calcium a. PTH increases reabsorption in the early distal tubule of the kidneys and mobilizes calcium from bone. b. Vitamin D (calcitriol) increases calefum reabsorption in the intestine and kidneys (ee above). ¢. Caleitonin, which is synthesized by C cells in the thyroid gland, inhibits osteoclasts, ‘hence inhibiting the release of calcium from bones. 4. Approximately 40% of calcium is bound to albumin, 13% to phosphorus and citrates, and 47% circulates as free, ionized calcium, which is metabolically active. (1) Alkalotic conditions decrease ionized calcium levels by increasing the amount of calcium bound to albumin: an alkalotic state increases the number of negative charges on albumin (less hydrogen ions), hence proportionately more calcium is ‘bound to albumin at the expense of the ionized level. (2) Hypoalbuminemia decreases the total serum calcium without altering the ionized calcium level. Causes of hypocalcemia a. Hypoalbe:minemia: most common non-pathologic cause b. Hypomagnesemta: most common pathologic cause; see below ¢. Vitamin D deficiency: see above 4. Primary hypoparathyroidism Causes of hypercalcemia a. Primary hyperparathyroldism: most common cause in the ambulatory population b. Malignancy: most common cause in a hospitalized patient ¢. Sarcoidosis: granuloma synthesis of vitamin D D. Phosphorus 1. 2. Itis the most abundant intracellular fluid anion. ‘Most foods contain phosphorus. 31(Note: This material is copyrighted. AN rights reserved. Edward Goljan, M.D. 2003 3. Funetions of phosphorous a, Mineralization of bones and teeth 6. Component of DNA and RNA. ©. Component of phosphorylated vitamins (e.g, thiamine, pyridoxine) and ATP d. Traps monosaccharides in cells (¢.g., phosphorylation of ghucose in glycolysis) ¢. Activates enzymes (€.g., protein kinase) and deactivates enzymes (e.g, glycogen synthase) £ Maintains pH: protons secreted into the renal tubule lumen react with dibasic phosphorus (HPO.") to form monobasic phosphorus (H:PO,), which is called titratable acidity. Control of phosphorous a. PTH has a phosphaturic effect. b, Vitamin D (calcitriol) increases reabsorption of phosphorus in the small bowel. 5. Causes of hypophosphatemia a. Respiratory and metabolic alkalosis: most common cause; alkalosis enhances glycolysis and phosphorylation of glucose b. Hypovitaminosis D due to malabsorption: decreased intestinal reabsorption of phosphorus c. Primary hyperparathyroidism: increased loss of phosphorus in urine 6. Causes of hyperphosphatemia a. Renal failure: most common cause; decreased renal excretion ’b. Primary hypoparathyroidism c. Normal children: higher levels of phosphorus help drive calcium into bone E. Sodium 1. Itis the most abundant extracellular fluid cation. 2. Itis primarily found in table salt. 3. Functions of sodium a. Regulation of pH, osmotic pressure, and water movement in body fluids + The serum sodium concentration is the primary determinant of water movements ‘by osmasis between the extracellular and intracellular fluid compartments. b. Maintains muscle and nerve excitability ©. Active transport of glucose, galactose, and amino acids in the small intestine 4. Maintains the diffusion potential of membranes, Control of sodium a. Diet b. Aldosterone: controls renat reabsorption (When present) and excretion (when absent) ©. Atrial natriuretic peptide: decreases reabsorption in the kidneys 5. Causes of hyponatremia a. Thiazide and loop diuretics: most common cause; increase rental excretion 'b. Inappropriate secretion of antidiuretic hormone: dilutional effect in plasma of excess ‘water reabsorption from the collecting tubules of the kidneys ©. Congestive heart failure and chronic liver disease: dilutional effect in plasma of excess water reabsorbed from the kidneys 6. Causes of hypernatremia 2. Osmotic diuresis: most common cause; loss of a hypotonic salt solution in the kidneys due to glucosuria, excess urea, or mannitol (treatment of cerebral edema) 'b, Diabetes insipidus: loss of watet due to deficiency or dysfunction of antidiuretic hormone 7. Clinical findings associated with hyponatremia and bypematremia: see Table ¥, Chloride 1. Itis the most abundant extracellular fluid anion. 32Note: This material is copyrighted. AM rights reserved. Edward Goljan, M.D. 2003 Sources of chloride include meats, fruits and vegetables, legumes, and nuts. Functions of chloride a. Regulation of pH and osmotic pressure b. Regulates neuromuscular excitability and muscle contraction Control of chloride a. Diet b, Aldosterone: controls renal reabsorption (when present) and excretion (when absent) Causes of hypochloremia a. Thiazide and loop diuretics: increase renal excretion b. Vomiting: lost ia the vornitus Causes of hyperchloremia a. Mineralocorticoid excess: increases both sodium and chloride reabsorption 'b. Renal tubular acidosis and diarrhea: loss of bicarbonate causes an increase in chloride to offset the loss of negative charges G. Potassium L 2 3. Itis the most abundant intracellular cation. Sources of potassium include meats, vegetables and fruits, nuts, and legumes. Functions of potassium a, Regulation of pH and osmotic pressure 'b. Regulation of neuromuscular exeitability and muscle contraction ©. Regulates insulin secretion: hypokalemia inhibits insulin secretion, while hyperkalemia stimulates insulin secretion Control of potassium a. Aldosterone: controls renal reabsorption (when absent) and excretion (when present) b. Arterial pH (1) Alkalotic conditions cause hydrogen ions to move out of the cell (provides protons) and potassium into the cell (leads to hypokalemia) in order to maintain electroneutratity.. (2) Acidotic conditions cause hydrogen ions to move into cells (for buffering) in ‘exchange for potassium (leads to hyperkalemia). ‘Causes of hypokalemia a. Thiazide and loop diuretics: increase renal excretion b. Vomiting and diarrhea: lost in the body fluids ¢. Aldosterone excess: increased renal excretion Causes of hyperkalemia a. Renal failure: MCC; decreased excretion of potassium 1b, Addison's disease: due to loss of aldosterone H, Magnesium 1 2. ‘Sources of magnesium include green vegetables, nuts, and legumes. Functions of magnesium a. Caleium metabolism: * Cofactor of adenylate cyclase involved in the activation of PTH Increases PTH synthesis and release b. Component of bone ¢. Muscle contraction: modulates the vasoconstrictve effects of intracellular calcium 4. Nerve impulse propagation ©. Cofactor in ATPases (¢.g., sodium/potassium pump ATPase, calcium pump ATPase) Causes of hypomagnesemia a. Alcoholism: most common cause; increased renal loss bb. Diuretics: increased renal loss ©. Drugs (eg., aminoglycosides, cisplatinum): increased renal loss 33Note: This material is copyrighted. All rights reserved. Edward Goljan, M.D. 2003 Caleium Phosphorous Sodium Potassium Magnesium Causes of hypermagnesemia a. Renal failure: decreased excretion b. Treatment of eclampsia with magnes Signs and symptoms of deficiency ‘+ Tetany (hypocalcemia lowers muscle and nerve threshold potential), © signs tetany: carpopedal spasm (thumb adducts into palm), Chvostek’s sign (facial twitch after taping VII nerve), muscle twitching, osteoporosis (decreased bone tass). “Muscle weakness, « rhabdomyolysis with myoglobinuria (due to low jum sulfate Signs and symptoms of excess + Kidney stones (calcium oxalate and phosphate), * metastatic calcification Galci-fication of normal tissues, eg., deposition in the kidneys is called nephrocalcinosis), © polyuria (due to calcification of renal tubule basement ‘memabranes). ‘* Hypocalcemia (high phosphorous drives calcium into bone and soft tissue), « ATP), « hemolytic anemia (due to low ATP). © Mental status abnormalities (cerebral edema, water shifts into cells by osmosis), # convulsions. ‘+ Muscle weakness (cannot repolatize muscle), polyuria (renders collecting tubule resistant to antidiuretic hormone). ‘+ hypocalcemia with tetany (acquired hypoparathyroidism due to impaired PTH secretion and resistance in target tissue), # tachycardia, hypovitaminosis D (inhibits activity of 1- hydroxylase). + Mental status abnormalities (intracellular shrinkage of neuroglial cells and neurons), ¢ convulsions. + Heart stops in diastole (must protect heart with injection of calcium gluconate). + Neuromuscular depression (depressed deep tendon reflexes, muscle weakness), « bradycardia. & Trace elements A. Tron 1. 2 Primary sources of iron include meat, eggs, vegetables, and fortified cereals. Tron is the structural component of heme in hemoglobin, myoglobin, ané the ‘eytochrome oxidase system. It is also an important cofactor for enzymes (¢-£., catalase). a. Meat has heme iron, which is ferrous and ready for reabsorption in the duodenum. * Once reabsorbed by duodenal enterocytes, heme is enzymatically degraded to release iron. Most of the iron is diverted to storage as apoferritin in the enterocyte, while a ‘small amount is delivered to plasma transferrin, the circulating binding protein of iron, Plants contain non-heme iran, which is in the ferric state, hence reabsorption is ‘more complex and involves a number of different binding-proteins before it is transferred to transferrin. Ferritin, a soluble iron-protein complex, isthe storage form of iron in the intestinal ‘mucosa, liver, spleen, and bone marrow. (1) Serum ferritin levels reflect iron stores in the bone marrow. (2) Serum ferritin is the best screening test for iron deficiency and iron overload disorders (c.g., hemochromatosis). Hemosiderin is an insoluble storage product of ferritin degradation, (1) Hemosiderosis is an acquired accumulation of hemosiderin in macrophages in tissues throughout the body: alcoholics and patients with a chronic transfusion requirement are at risk for hemosiderosis b. 34‘Note: This material is copyrighted. All rights reserved, Edward Goljan, M.D. 2003 (2) Hemochromatosis is an AR disease characterized by unrestricted reabsorption of iron from the duodenum leading to an accumulation of iron in liver, heart, pancreas, skin, and other tissues. @) Either condition can cause cirrhosis of the liver, bronze skin color, diabetes mellitus, malabsorption, and heart failure. ¢. Transferrin, the primary binding-protein for iron, is synthesized transports iron to macrophages in the bone marrow for storage or to the developing RBCs for Hb synthesis. (1) When iron stores in the bone marrow macrophages are decreased (e.g. iron deficiency), liver synthesis of transferrin increases, which increases total iron- binding capacity. 2) When iron stores in the bone marrow macrophages are increased (eg, hemochromatosis), transferrin synthesis is decreased, which deereases total iron- binding capacity. 3. Causes of iron deficiency vary by ag ‘Newborn: bleeding Meckel’s diverticulura Child: bleeding Mecke!'s diverticulum, milk diet ‘Woman < 50-years-old: menorthagia ‘Man < 50-years-old: peptic ulcer disease Men and women over 50-years-old: colon polyp or cancer ‘auses of excess serum iron Iron poisoning: common in children; causes hemorrhagic gastritis and liver necrosis Iron overload diseases: hemochromatosis (See above), hemosiderosis (see above), sideroblastic anemia (due to pyridoxine deficiency, lead poisoning, alcoholism) ve goreose the liver and ‘* Sideroblastic anemias are associated with excess iron accumulation in ‘mitochondria owing to problems in heme synthesis. ‘© Excess iron in mitochondria produces ringed sideroblasts (mitochondria are located around the nucleus of immature RBCs), B. Zine 1. Sources of zinc include meat, liver, eggs, and oysters. 2. Zine primarily serves as a cofactor for metalloenzymes. a. Superoxide dismutase b. Collagenase: important in remodeling of a wound and replacing type III collagen with type I collagen to increase tensile strength c. Alcohol dehydrogenase: converts alcohol into acetaldehyde 4. Alkaline phosphatase: important in bone mineralization; marker of obstruction to bile flow in the liver or common bile duct 3. Zine is also important in spermatogenesis and growth in children. 4, Causes of zine deficiency a, Miscellaneous diseases: alcoholism, rheumatoid arthritis, acute and chronic inflammatory diseases, chronic diarrhea b. Acrodermatitis enteropathica: AR disease associated with a dermatitis, diarrhea, growth retardation in children, decreased spermatogenesis, and poor wound healing C. Copper 1. Sources of copper include shellfish, organ meats, poultry, cereal fruits, and dried beans. 2. Copper primarily serves as a cofactor for metalloenzymes. Ferroxidase: binds iron to transferrin; causes iron deficiency if deficient LLysyl oxidase: cross-linking of collagen and clastic tissue Superoxide dismutase: neutralizes superoxide, an O; free radical ‘Tyrosinase: important in melanin synthesis; deficient in albinism Cytochrome ¢ oxidase: component of ETC saose 35Note: This material is copyrighted. All rights reserved. Edward Goljan, M.D. 2003 Ceruloplasaiin, a copper-binding plasma protein that is synthesized in the liver, is involved in copper transport and regulation. Copper deficiency (hypocupremia) is most often due to total parenteral nutrition fin excess of fee copper (bypereapreml) ie preset in Wions dase an AR disease associated with a defect in secreting copper into bile. ‘© Characteristic findings include chronic liver disease, deposition of free copper into the eye (Kayser-Fleischer ring) and lenticular nuclei (dementia, movement disorder), low serum ceruloplasmin, and high free copper levels in blood and urine. D. Iodine if 2 3. Sources of iodine include iodized table salt and seafood. Iodine is used in the synthesis of thyroid hormones. Todine deficiency is primarily due to an inadequate intake of seafood or iodized table salt, E. Chromium ic 2. 3 4 Sources of chromium include wheat germ, liver, and Brewer's yeast. Itis a component of glucose tolerance factor, which maintains a normal glucose, and is also a cofactor of insulin, which facilitates its binding to adipose and muscle. Chromium deficiency primarily occurs in patients receiving TPN. Clinical findings associated with chromium deficiency: see Table F. Selenium 1 2. 3. Sources of selenium include seafood and liver. It is 2 component of glutathione peroxidase, which converts oxidized glutathione into reduced glutathione in the pentose phosphate pathway. © Glutathione is a potent antioxidant that neutralizes peroxide and peroxide free radicals. Selenium deficiency is primarily occurs in patients receiving TPN. G. Fluoride Tron, Zine Copper Iodide Chromium Selenium Fluoride Sources of fluoride include tea and fluorinated water. tis a structural component of calcium hydroxyapatite in bone autd teeth (sec above). Deficiency of fluoride is primarily due to inadequate intake of fluoridated water. ‘An excess in fluoride is primarily due to an excess of flucride in drinking water. * Fluorosis is associated with chalky deposits on the tecth, calcification of ligaments, and an increased risk for bone fractures, ‘+ Microcytie anemia (iron deficiency is the most common cause), + Yow serum ferritin, « low serum iron, * high total iron binding capacity (correlates with decreased transferrin synthesis), © Plummer-Vinson syndrome (esophageal webs, glossitis, spoon nails, achlorhydria), « excessive fatigue. ‘* Poor wound healing, * dysgeusia (cannot taste), « anosmia (cannot smell), * perioral rash, hypogonadism, * growth retardation. * Microcytic anemia (decreased ferroxidase activity), * dissecting aortic aneurysms (decreased cross-bridging of elastic tissue), « poor wound healing * Goiter (due to relative or absolute deficiency of thyroid hormones). ‘Impaired glucose tolerance, + peripheral neuropathy. ‘ Muscle pain and weakness, « cardiomyopathy. ‘= Dental caries. 36Note: This material is copyrighted. All rights reserved. Edward Goljan, MD. 2003 & Locations of biochemical processes in cell: + cytosol: glycolysis, pentose phosphate shunt, FA synthesis, glycogen synthesis, * mitochondrial matrix: B-oxidation of FAs, TCA cycle, inner ‘mitochondrial membrane: oxidative phosphorylation, * both cytosol and mitochondria: ‘gluconeogenesis, urea cycle, heme synthesis © — Comparison Chart of the Well Fed State, Fasting State, and Starved State: recall that insulin is, the key hormone of the fed state and glucagon of the fasting state ‘Well fed state Fasting state Starved state Glycogenesis Increased ‘None None Glycogenolysis, Decreased: none in Increased: early supply None: glycogen used up the liver, some in of glucose derived from muscle liver not muscle Gluconeogenesis None Increased: primary Decreased: just enough to source of glucose after supply RBCs slycogenolysis ‘Triacylglycerol synthesis Increased None None in liver/adipose Lipolysis None Increased Increased Fate of glycerol Synthesize more Substrate for Substrate for triacylglycerol in liver gluconeogenesis gluconeogenesis Broxidation of fatty acids None Inereased Markedly increased: primary fuel for muscle Musele catabolism ‘None: increased Tnereased: supply amino Decreased: conserve protein synthesis and acids for ‘muscle for important body uptake of amino acids ghiconeogenesis functions Urea synthesis/excretion Remains constant: Increased: deamination Decreased: less muscle handles NH, load of amino acids used for breakdown of protein with from protein sluconeogenesis less amino acids to degradation in gut by increases urea synthesis degrade bacteria Ketone body synthesis. None Increased ‘Markedly increased: by- product of acetyl CoA from increased B- oxidation of fatty acids ‘Muscle use of glucose for Primary fuel Decreased None: mainly uses fatty fuel acids ‘Muscle use of fatty acids None Increased: primary fuel Markedly increased: for fuel primary fuel Masele use of ketones None Some: alternative fuel None: allows the brain to for fuel uuse ketones for fuel Brain use of glucose for Remains constant Remains constant Decreased: allows RBCs fuel to primarily use glucose for fuel Brain use of ketones for None None Increased: primary fuel fuel RBC use of glucose for Remains constant Remains constant Remains constant fuel © — Rate limiting reaction in cholesterol synthesis: * HMG Co reductase: inhibited by cholesterol and statin drugs (competitive inhibition with mevalonic acid), ¢ enzyme converts HMG CoA into mevalonic acid 37‘Note: This material is copyrighted. All rights reserved. Edward Goljan, M.D. 2003 Rate limiting reaction in fatty acid synthesis: « acetyl CoA carboxylase, © enzyme converts acetyl CoA into malonyl CoA Rate limiting reaction in B-oxidation of fatty acids: + camitine acyltransferase I, # enzyme in ‘outer membrane of mitochondria removes acyl group from fatty acyl CoA and transfers it to carnitine (ecylcamitine), « inner membrane enzyme removes acyl group from acyleamitine and transfers it back to CoA to produce fatty acyl CoA Rate limiting reaction in glycolysis: * phosphofructokinase I, © enzyme converts fructose 6- .6-bisphosphate Rate limiting reaction in gluconeogenesis: + fructose 1,6-bisphosphatase, ¢ enzyme converts fructose 1,6-bisphosphate to fructose 6-phosphate Rate limiting reaction in glycogen synthesis: + glycogen synthase, * enzyme forms a-l,4 glycosidic linkages between a glucose unit from UDP-glucose and the non-reducing end of an existing glycogen chain Rate limiting reaction in glycogenolysis: « liver phosphorylase, * enzyme cleaves a-1,4 linkages (Feleases glucose I-phosphate) but stops working four glucose units from the branch point Rate limiting reaction in pentose phosphate pathway: « glucose 6-phosphate dehydrogenase, « ‘GOPD converts glucose 6-phosphate to 6-phosphogluconate Rate limiting reaction in lipolysis: * hormone sensitive lipase, * enzyme hydrolyzes triacylglycerol into fatty acids and glycerol Rate limiting reaction in urea cycle: * carbamyl phosphate synthase 1, ¥ enzyme converts CO; + NH, +2 ATP into carbamyl phosphate Rate limiting reaction in pyrimidine metabolism: ¢ carbamyl phosphate synthase II, « enzyme ‘converts 2 ATP + CO; + glutamine into carbamoy\ phosphate Rate limiting reaction im purine metabolism: * glutamine-PRPP aminotransferase, * enzyme converts PRPP + glutamine into 5'-phosphoriboysyamine Rate limiting reaction in heme synthesis: « ALA synthase, ¢ enzyme converts succinyl CoA + glycine into 8-aminolevutinic acid Rate limiting reaction in ketone body synthesis: « HMG CoA synthase, converts acctoacetyl CoA into HMG CoA Leseh Nyhan: © SXR with absent HGPRT, « self-mutilation, « hyperuricemia, + mental retardation Branched chain amino acids and maple syrop urine disease: * only muscle can metabolize branched chain amino acids, « missing dehydrogenase enzyme Funetions of cholesterol: vitamin D synthesis in the skin (7-dehydrocholesterol), © steroid synthesis, « cell membranes, « synthesis of bile salts/acids ‘Acetyl CoA ases: ¢ FA synthesis, # CH synthesis, « ketone body synthesis, « synthesis of citrate along with oxaloacetic acid, « uot a substrate for gluconeogenesis Ketone body synthesis: « HMG CoA synthasc is the rate-limiting enzyme, « HMG CoA lyase converts HMG CoA into acetoacetic acid, while in cholesterol synthesis, HMG CoA reductase converts HMG CoA into mevalonic acid, « 23 ATP produced for fuel Enzyme kinetics: * Vax represents the maximum velocity of an enzyme reaction when all enzyme sites are fully saturated, « K,, (Michaelis-Menton constant) of an enzyme represents the substrate concentration at which the reaction velocity is one-half of Vmax (Vinax/2), « Km describes the enzymes affinity for substrate, increased Km indicates decreased affinity of the enzyme for substrate, « decreased K,, indicates increased affinity of the enzyme for substrate Competitive inhibitors: * Vax is not changed since the reaction rate is unchanged whether the competitive inhibitor (e.g, ethylene glycol, methyl alcohol) is binding to the active site of the enzyme (e.g. alcohol dehydrogenase) of alcohol, * Ky increases (decrease in the affinity of the ‘enzyme for alcohol), since the enzyme is also actively binding with ethylene glycol/methyl alcohol, * infusing alcohol decreases the metabolism of ethylene glycol/methy! alcohol, hence the effect of a 38Note: This material is copyrighted. All rights reserved. Edward Goljan, M.D. 2003 inhibitor is reversed by increasing substrate, + methotrexate is also a competitive = Reversible noncompetitive inhibitors: * Vea is decreased, however the Kq remains the same, * reversible noncompetitive inhibitors bind reversibly (non-covalent bonds) away from the active binding site of the enzyme and form either unreactive enzyme-inhibitor complexes or enzyme- substrate-inhibitor complexes, + affinity of the enzyme for substrate is unchanged (K,), since the active binding site is unaltered and normally binds with substrate, * Vaux is decreased since the inhibitor inactivates the enzyme, which automatically decreases the effective concentration of active enzyme, « increasing substrate concentration does not reverse the effect of the inhibitor, since the inhibitor is blocking enzyme activity away from the active binding site, * examples of reversible noncompetitive inhibitors include angiotensin converting enzyme inhibitors (substrate is angiotensin 1); physostigmine, which is @ cholinesterase inhibitor (substrate acetylcholine); and, allopurinol, an xanthine oxidase inhibitor (substrate xanthine), permanently inactivate enzymes by forming strong covalent bonds: examples include lead (rveversibly inhibits ferrochelatase and 8-aminolevulinic acid dchydrase), aspirin (irreversibly inhibits platelet cyclooxygenase), and organophosphates (irreversibly inhibit cholinesterase), enzyme kinetics are the same as those for reversible noncompetitive inhibitors = Urea cycle: + method of eliminating ammonia, « located in the hepatocyte, « chronic liver disease: Tow BUN, elevated ammonia © — Cholesterol synthesis: first few steps are similar to ketone body synthesis except HMG CoA lyase is used instead of HMG CoA reductase = Apolipoprotein 100 (liver) and 48 (intestine) © Liver can not use ketones for fuel: liver cannot activate acetoacetate in the mitochondria, which requires succinyl CoA: acetoacetate CoA transferase (a thiotransferase enzyme) in order to convert AcAc into acetoacetyl CoA. = Insulin: « key hormone of the fed state, * activates phosphatase (dephosphorylation) = — Glucagon: ¢ key hormone of the fasting state, « activates protein kinase (phosphorylation) Major source of NADPH: « pentose phosphate shunt, malate dehydrogenase (malic enzyme) reaction to a lesser extent, « NADPH supplies reducing equivalents = — Mechanism of ketoacidosis in DKA: increased -oxidation of fatty acids and production of acetyl CoA, which is used by the liver to synthesize ketone bodies Energy in cardiac muscle: f-oxidation of fatty acids Lipid faets: « functions of HDL: reservoir for apolipoproteins in the blood; transport of esterified cholesterol to the liver; transfers cholesterol esters to VLDL in exchange for triglyceride using cholesterol ester transport protein; taken up by scavenger receptors in the liver, « in patients with diabetic ketoacidosis you would expect... activation of hormone sensitive lipase in the adipose, conversion of glycerol 3-phosphate into dihydroxyacetone phosphate, « in the fasting state, you ‘would expect: loss of inhibition of carnitine acyltransferase by malonyl-CoA, « in the fed state, you would expect: activation of citrate lyase in the cytosolsinactivation of hormone sensitive lipase in adipose; inereased production of palmitic acid in the cytosol = Findings in PKU: + AR disease with deficiency of phenylalanine hydroxylase, * newborn must be exposed to phenylalanine (PHY) in the diet before it is increased, * mousy odor, * projectile vomiting simulating congenital pyloric stenosis, « tyrosine missing, hence it must be supplied in the diet, + can diagnose by amniocentesis and finding the abnormal gene, « eliminate phenylalanine from diet: Nutrasweet is aspartate and phenylalanine, s0 cannot use it, woman with PKU who is pregnant must be on a PHY free diet: affected fetus will develop permanent CNS damage in utero owing to exposure of PHY isorders of galactose metabolism: + galactose derives from lactose metabolism: lactose + lactase» glucose + galactose, + galactose metabolism in sequence is as follows: galactose + a4 39‘This material is copyrighted. All rights reserved. Edward Goljan, M.D. 2003 galactokinase —» galactose 1-PO,, + galactose 1-PO, + GALT (galactose-I-phosphate uridyl transferase) + UDP-glucose-> glucose 1-PO, + UDP-ealactose, * glucose I-PO, + phosphoglucomutase —> glucose 6-PO, (6 carbon intermediate) + glucose 6-PO« + glucose 6- phosphatase (gluconeogenic enzyme)-> glucose, * galactokinase deficiency: benign AR disease; positive urine Clinitest: Clivitest detects all reducing sugars except sucrose, which is not a reducing sugar, « galactosemia: AR disease with deficiency of GALT; excess galactose is converted into the galactitol (polyol or alcohol sugar), which, like sorbitol, is osmotically active: damages lens, nerve tissue, CNS, liver; exeess galactose 1-PO, is toxic: cirrhosis, mental retardation, renal damage, neonatal hypoglycemia (ack af glucose 6-PO,, & substrate for gluconeogenesis); Rx is a lactose free diet for the first two yrs; pregnant women with galactosemia can synthesize lactose in their breast milk via the following reactions: UDP-glucose + UDP-hexase epimerase —> UDP- galactose, UDP-galactose + lactase synthetase-» lactose + UDP Disorders in fructose metabolism: * fructose metabolism in sequence: sucrose + sucrase > glucose + fructose; fructose + fructokinase —» fructose 1-PO,; fructose 1-PO, + aldolase B > glyceraldehyde 3- phosphate + dihydroxyacetone phosphate (DHAP, both are 3 carbon intermediates that are gluconeogenic substrates), « fructose can be synthesized from mannose (and vice versa) and sorbitol, * fructose is an essential nutrient for sperm stored in the seminal vesicles, « essential fructosuria: AR disease with missing fructokinase; positive urine Clinitest due to fructose, ¢ hereditary fructose intolerance: AR disease with a deficiency of aldolase B; accumulation af fructose I-PO,, which is toxic to the liver (cirrhosis); fasting hypoglycemia due to a decrease in 3 carbon intermediates for gluconeogenesis; severe hypophosphatemia: excess fructose traps phosphate in cells, depletion of ATP leads to RBC hemolysis and rhabdomyolysis, increased adenosine monophosphate (AMP), which is a purine, acid leading {0 gout, must eliminate table sugar (sucrose) from the diet Sorbitol: « osmotically active solute that is synthesized in those tissue containing aldose reductase, which include: lens, seminal vesicles (note in the biochemical reaction listed below how glucose is ‘converted into fructose in the seminal vesicles), Schwann cells, retina, kidneys, # aldose reductase converts ghicose into sorbitol and sorbitol dehydrogenase converts sorbitol into fructose: NADP*NAD' NADH; Glucose Sorbitol _———p Fructose aldose reductase sorbitol dehydrogenase ‘* hyperglycemic states, like diabetes mellitus, there is an excess of sorbitol produced in the above tissues leading to osmotic damage: cataracts, peripheral neuropathy due to destruction of Schwann cells, microaneurysms in retinal vessels due to destruction of pericytes Homocystinuria: « AR disease with a deficiency of cystathionine synthase, * metabolism of homocystine in sequence is as follows: methionine + ATP —> S-adenosylmethionine (SAM) + methyl acceptors + methyliransferase -> S-adenosylhomocysteine + methylated products (donates methyl groups for 1 carbon transfers); S-adenosythomocysteine + H;0 —> homocysteine + adenosine; homocysteine + serine + eystathionine synthase —» cystathionine, + in homocystinuria, both homocysteine and methionine are increased in serum, * S/S of homocystinuria that resemble Marfan syndrome (example of genetic heterogeneity): dislocated lens, arachnodactyly, eunuchoid, * distinctive features of homocystinuria: increase in plasma homocysteine levels leads ‘to vescel damage/thrombosis (strokes, AMI), mental retardation, « increased urine homocysteine and increased serunVurine methionine Alcaptonuria and hereditary tyrosinosis: © metabolism of phenylalanine is as follows: phenylalanine + phenylalanine hydroxylase. (deficient in PKU)-> tyrosine; tyrosine > 4- hhydroxyphenyl pytuvate > homogentisate; homogentisate + homogentisate oxidase (deficient in alkaptonuria) > maleylacetoacetate; maleylacetoacetate + fumarylacetoacetate hydrolase (deficient in hereditary tyrosinemia) > fumarylacetoacetate; fumarylacetoacetate —> acetoacetate + 40‘Note: This material is copyrighted. All rights reserved. Edward Goljan, M.D. 2003 fumarate (present in TCA cycle), « alkaptonuria: AR disease with an absence of homogentisate ‘oxidase; accumulation of black, homogentisate pigment in joints/cartilage leads to degenerative Joint disease; urine turns black when oxidized upon exposure to light, « hereditary tyrosinosis: ‘AR disease with a deficiency of fumarylacetoacetate hydrolase; increase in serum tyrosine: cabbage-like odor, cirthosis with an extremely high incidence of hepatocellular carcinoma, renal disease (aminoaciduria), death in the first yr of life = Lysosomal storage diseases: # definition: absence of degrading enzymes in lysosomes: contain hydrolytic enzymes; accumulation of complex substrates in lysosome: sphingolipids, glycos- aminoglycans, glycogen (Pompe's disease), + most are AR diseases with the exception of two diseases, which are SXR: Fabry’s disease, Hunter's disease, = —Glycosaminoglyeans (GAGs) and their disorders: © GAGs:complexes of predominantly branched, strongly negatively charged polysaccharide chains with repeating units of amino sugars (D-glucosamine or D-galactosamine) and acid sugars (L-iduronic acid or D-glucuronie acid), « chondroitin sulfate: most abundant GAG, important component in cartilage, © heparan sulfate: mainly responsible for the negative charge of the glomerular basement membrane, * heparin— anticoagulant, © Keratan sulfate, » hyaluronic acid: major component of synovial fluid (joint lubricant), « dermatan sulfate: ground substance in heart valves that is increased in mitral valve prolapse, increased in pretibial myxedema, Hurler’s disease: AR disease with a deficiency of ‘G-I-iduronidase; accumulation of dermatan/heparan sulfate; severe mental retardation; coarse facial features; comeal clouding; coronary artery disease: lipid accumulates in coronsry vessels; vacuoles in peripheral blood leukocytes, « Hunter's disease: SXR disease with a deficiency of L~ iduronosulfate sulfatase; accumulation of dermatan/heparan sulfate; milder disease than Hurler's % —Sphingolipids and their disorders: + sphingolipids include: sphingomyelin, cerebrosides, gangliosides, + sphingomyelin: involved in the synthesis of cell membranes in nerve tissue; sphingosine is the backbone of sphingomyelin; sphingosine is used to produce ceramides: sphingosine + fatty acids -> ceramide, * ceramide + phosphorylcholine-> sphingomyelin, © ceramide + glucose or galactose—> gluco- or galactocerebrosides, « ceramide + oligosaccharides -> gangliosides, « Tay-Sachs disease: AR with a deficiency of hexosaminidase (o-subunit ‘nucleotide insertion leads to a frameshift mutation and an abnormal hexosaminidase; accumulation of GM; ganglioside; common in Ashkenazi Jews; normal at birth» severe mental retardation by 6 months; blindness with a cherry red spot in the macula; mo hepato- splenomegaly; electron microscopy exhibits whorled configurations in lysosomes that look exactly the same as lamellar bodies with surfactant in type TI pneumocytes, # Niemann-Piek: AR disease with a deficiency of sphingomyelinase; accumulation of sphingomyelin: bubbly appearance in ‘macrophages! neurons; mental retardation; hepatosplenomegaly; EM exhibits zebra bodies in lysosomes: look like zebra stripes, * Gaucher disease: AR disease with deficiency of glucocerebrosidase; accumulation of glucocerebroside: fibrillary appearance (crumbled up newspaper) in macrophages; adult type associated with massive hepatosplenomegaly and an {increase in serum total acid phosphatase derived from macrophages, » metachromatic leukodystrophy: AR discase with a deficiency of arylsulfatase A and accumulation of sulfatide: results in the synthesis of abnormal myelin; sulfatides stain positive with metachromatic stat peripheral neuropathy; wrine arylsulfatase activity decreased/absent, ¢ Krabbe disease: AR disease with a deficleney of galactosyleeramidase and accumulation of galactocerebroside: results in the synthesis of an abnormal myelin; progressive psychomotor retardation; multinucleated globoid cells (histiocytes) in CNS, + Fabry disease: SXR disease with a deficiency of o- galactocerebrosidase A and accumulation of ceramide trihexoside; angiokeratomas on skin; hypertension; renal failure = Glycogen synthesis (glycogenesis): © glycogen synthesis (glycogenesis, occurs in the fed state) in sequence: glucose + glucokinase > G6-PO,; G6-PO, + phosphoglucomutase -> G1-POg GI-PO. 4
You might also like
Main Dip Step 2ck Notes Info Doc - Read Only
PDF
No ratings yet
Main Dip Step 2ck Notes Info Doc - Read Only
573 pages
Ini May 2025 Recall
PDF
No ratings yet
Ini May 2025 Recall
22 pages
Goljan Notes
PDF
No ratings yet
Goljan Notes
403 pages
MM Pharma Merged
PDF
No ratings yet
MM Pharma Merged
210 pages
Step 2 CK
PDF
No ratings yet
Step 2 CK
10 pages
Nbme 7
PDF
80% (5)
Nbme 7
205 pages
Veterinary Hematology: A Diagnostic Guide and Color Atlas. 1st Edition. ISBN 1437701736, 978-1437701739
PDF
100% (27)
Veterinary Hematology: A Diagnostic Guide and Color Atlas. 1st Edition. ISBN 1437701736, 978-1437701739
23 pages
Emma Holliday Peds Review PDF
PDF
100% (2)
Emma Holliday Peds Review PDF
78 pages
Goljan - High Yield Questions For USMLE
PDF
100% (1)
Goljan - High Yield Questions For USMLE
36 pages
Goljan Pathology Notes For Step 2 USMLE PDF
PDF
75% (4)
Goljan Pathology Notes For Step 2 USMLE PDF
446 pages
Medical Ethics
PDF
100% (1)
Medical Ethics
13 pages
HY Mixed USMLE Review Part IV
PDF
100% (1)
HY Mixed USMLE Review Part IV
22 pages
Nbme Notes
PDF
No ratings yet
Nbme Notes
4 pages
Golijan, S Audio Lec Notes
PDF
100% (6)
Golijan, S Audio Lec Notes
186 pages
Usmle Super
PDF
No ratings yet
Usmle Super
239 pages
High Yield Biochemistry-Goljan Note For Step 1
PDF
100% (1)
High Yield Biochemistry-Goljan Note For Step 1
12 pages
Alternative NBME Vocabulary
PDF
No ratings yet
Alternative NBME Vocabulary
3 pages
CMS Peads 1 & 2 Answers
PDF
100% (1)
CMS Peads 1 & 2 Answers
12 pages
Goljan Step 1 Questions
PDF
0% (1)
Goljan Step 1 Questions
3 pages
Goljan Step 2 Notes PDF
PDF
100% (2)
Goljan Step 2 Notes PDF
446 pages
100 High Yield NBME
PDF
No ratings yet
100 High Yield NBME
3 pages
Biostatistics Lecture 3
PDF
No ratings yet
Biostatistics Lecture 3
18 pages
HY USMLE Review Part II
PDF
No ratings yet
HY USMLE Review Part II
18 pages
Uworld CK Questions Notes
PDF
No ratings yet
Uworld CK Questions Notes
2 pages
Integrated Pathology Notes For USMLE Step - 2 - Edward Goljan - 2003 PDF
PDF
100% (1)
Integrated Pathology Notes For USMLE Step - 2 - Edward Goljan - 2003 PDF
446 pages
Chemical Pathology Workshop II - Diagnostic Theory in Chemical Pathology (2017.11.14)
PDF
100% (1)
Chemical Pathology Workshop II - Diagnostic Theory in Chemical Pathology (2017.11.14)
57 pages
UW Notes - 6 - Public Health
PDF
No ratings yet
UW Notes - 6 - Public Health
27 pages
Goljan General Pathology Notes PDF
PDF
100% (3)
Goljan General Pathology Notes PDF
233 pages
Concepts in Chemical Pathology
PDF
No ratings yet
Concepts in Chemical Pathology
41 pages
General Pathology
PDF
100% (1)
General Pathology
233 pages
My Divine Intervention Podcast Daily
PDF
No ratings yet
My Divine Intervention Podcast Daily
1 page
HY Endocrine Usmle
PDF
No ratings yet
HY Endocrine Usmle
22 pages
Anatomy High Yield From Usmleworld Forum
PDF
No ratings yet
Anatomy High Yield From Usmleworld Forum
9 pages
HY USMLE Review Part III
PDF
100% (1)
HY USMLE Review Part III
16 pages
Uworld Flash Cards 2
PDF
No ratings yet
Uworld Flash Cards 2
6 pages
New Goljan Notes
PDF
100% (1)
New Goljan Notes
446 pages
Goljan High Yield Facts
PDF
100% (1)
Goljan High Yield Facts
36 pages
Satish Advani 271 CK Experience
PDF
No ratings yet
Satish Advani 271 CK Experience
10 pages
Stitelman's Surgery Shelf Review: Edmund - Bartlett@uphs - Upenn.edu
PDF
No ratings yet
Stitelman's Surgery Shelf Review: Edmund - Bartlett@uphs - Upenn.edu
57 pages
Combank Notes
PDF
No ratings yet
Combank Notes
7 pages
USMLE Step 1 Notes
PDF
100% (1)
USMLE Step 1 Notes
7 pages
Sample Step 2 Study Schedule PDF
PDF
No ratings yet
Sample Step 2 Study Schedule PDF
2 pages
Diagnostic-Test 63
PDF
No ratings yet
Diagnostic-Test 63
40 pages
UWorld Notes - Mark Chaskes
PDF
No ratings yet
UWorld Notes - Mark Chaskes
36 pages
Veterinary Hematology ISBN 1437701736, 9781437701739 Complete PDF Download
PDF
No ratings yet
Veterinary Hematology ISBN 1437701736, 9781437701739 Complete PDF Download
15 pages
Surgery Uworld
PDF
No ratings yet
Surgery Uworld
2 pages
Uworld Biostatistics
PDF
No ratings yet
Uworld Biostatistics
2 pages
Nbme 12 Block 4 Some Explanations
PDF
No ratings yet
Nbme 12 Block 4 Some Explanations
14 pages
RX Beta Blocker Posioning: Recent Ingestion - Emesis, Bradycardia: Atropine - Isoproterenol - Glucagon - Pace
PDF
100% (2)
RX Beta Blocker Posioning: Recent Ingestion - Emesis, Bradycardia: Atropine - Isoproterenol - Glucagon - Pace
13 pages
Goljan Pathology Notes For Step 2 USMLE PDF
PDF
No ratings yet
Goljan Pathology Notes For Step 2 USMLE PDF
446 pages
1-3, Diagnostic Accuracy of Laboratory Tests
PDF
No ratings yet
1-3, Diagnostic Accuracy of Laboratory Tests
46 pages
Goljan Pathology Notes For Step 2 USMLE PDF
PDF
No ratings yet
Goljan Pathology Notes For Step 2 USMLE PDF
446 pages
SESSION 6. Assessing Value of Tests
PDF
No ratings yet
SESSION 6. Assessing Value of Tests
15 pages
General Pathology Notes PDF
PDF
No ratings yet
General Pathology Notes PDF
233 pages
Goljan - Integrated Pathology Notes PDF
PDF
No ratings yet
Goljan - Integrated Pathology Notes PDF
60 pages
GOLJAN General Pathology PDF
PDF
No ratings yet
GOLJAN General Pathology PDF
233 pages
Diagnostic Talk 2012
PDF
No ratings yet
Diagnostic Talk 2012
28 pages
Integrated Pathology Notes PDF
PDF
No ratings yet
Integrated Pathology Notes PDF
69 pages
Step 2 CK
PDF
No ratings yet
Step 2 CK
2 pages
Part 5 PDF
PDF
No ratings yet
Part 5 PDF
4 pages